You are on page 1of 48

DE LA SALLE UNIVERSITY MANILA

RVR COB DEPARTMENT OF ACCOUNTANCY


INTTHRY 3rd Term AY 16 - 17

Auditing Theory Prof. Francis H.Villamin

REVIEWER PART III

Code of Ethics for Professional Accountants in the Philippines

1. The Revised Code of Ethics for Professional Accountants in the Philippines independence
requirements suggest that a CPA should evaluate whether a particular threat to independence would
lead a reasonable person, aware of all the relevant facts to conclude that
a. A questioning mind reveals doubt as to independence.
b. An unacceptable risk of non-independence exists.
c. The accountant is definitely not independent.
d. There is substantial cause for a legal finding on non-independence.

2. ABC & Company CPAs has one office. Which of the following is least likely to impair independence
with respect to an audit client?
a. The client owes the firm for two prior year years audit fees.
b. A partner in the CPA firm is the son the president of the client.
c. The wife of a partner in the firm has an immaterial direct financial interest in the client.
d. A partner in the firm has an investment in a mutual fund that a has a direct interest in the client

3. The Revised Code of Ethics for Professional Accountants in the Philippines allows an auditor to
perform which of the following services for an audit client:
a. Performance of bookkeeping services for the client.
b. Authorization of transactions for the client.
c. Preparation of client source documents.
d. Preparation and posting of journal entries without clients approval.

4. A small CPA firm provides audit services to a large local company. Almost eighty percent of the CPA
firms revenues come from this client. Which statement is most likely to be true?
a. Appearance of independence may be lacking.
b. The small CPA firm is satisfactory if the auditor exercises due skeptical negative assurance care
in the audit.
c. The small CPA firm does not have the proficiency to perform larger audit.
d. The auditor should provide an emphasis of a matter paragraph to his/her audit report
adequately disclosing this information and then it may issue an unqualified opinion.

5. Which of the following statements is correct?


a. Client prepared records (e.g. the general ledger) may be retained by the CPA until fees due to
the CPA are received.
b. CPA working papers are the joint property of the CPA and the client
c. Supporting records not reflected in the clients records (e.g. proposed adjusting entries) may be
withheld by the CPA if the fees for the engagement remain unpaid.
d. CPA working papers that include copies of clients records are not available to third parties
under any circumstances.

6. Independence is required of a CPA performing


a. Audits, but not any other professional services.
b. All attestation services, but not other professional services.
c. All attestation and tax services, but not other professional services.
d. All professional services.

7. A CPA sole practitioner purchased stock in a client corporation and placed it in a trust as an
educational fund for the CPAs minor child. The trust securities were not material to the CPA but
were material to the childs personal net worth. Would the independence of the CPA considered to
be impaired with respect to the client?
a. Yes, because the stock would be considered a direct financial interest and consequently
materiality is not a factor.
b. Yes, because the stock would be considered an indirect financial interest that is material to
CPAs child.
c. No, because the CPA would not be considered to have a direct financial interest in the client.
d. No, because the CPA would not be considered to have a material indirect financial interest in the
client.
AT Reviewer Part III Page 2

8. On August 20, 2014, Benedict Diaz, CPA and partner was offered and accepted the engagement to
audit the annual financial statements of Alive Corporation for the fiscal and calendar years ended
December 31, 2014. The audit began September 15, 2014 and ended on March 17, 2015. Alive
Corporation is regulated by the SEC. Diaz served as controller of Alive Corporation from November
5, 2007 until January 12, 2014, at which time he terminated his employment with Alive.

Diaz owned a material amount of Alives ordinary shares from November 5, 2004 until August 15,
2014, at which time he sold the shares. Is Benedict Diaz in violation of the Code of Ethics due to
impairment of independence?
a. Yes, because Benedict Diaz owned Alive shares at the start of the audit for December 31, 2014.
b. Yes, because Benedict Diaz served as controller for Alive Corporation in previous years.
c. Yes, because Benedict Diaz had an employment relationship with the client during part of the
period covered by the financial statements.
d. Yes, because it is still less than a year since Benedict Diaz has owned shares in Alive
Corporation.

9. CPA, Carmelie Henson performs audit of the local ballet society. Because of her good work, she
was elected as honorary member of the board of directors. Carmelie will not be considered
independent unless:
a b c d
- The position is in fact purely honorary Yes Yes Yes Yes
- Listings of directors show she is an honorary member. No Yes Yes No
- She restricts participation strictly to the use of her name. Yes No Yes No
- She does not vote or participate in management functions. Yes Yes Yes Yes

10. Justine, CPA is in charge of the staff of Twins Resort, Inc. Four young members of the audit firms
professional staff are working with Justine on this engagement, all of which are avid divers. Twins
Resort owns two condominiums in Anilao Batangas, which it uses to primarily to entertain clients.
The controller of Twins Resort, Inc. has told Mr. Justine that his whole team is welcome to use the
condominiums at no charge any time that are not already in use. How should Justine, CPA respond
to this offer?
a. Justine should withdraw from the engagement due to a significant threat to independence.
b. Justine alone may accept the offer, but his staff may not use the condominiums.
c. Justine should decline the offer, both for himself and his staff.
d. Justine may accept the offer, but only in favor of his staff, he cannot use the condominiums
because as partner, Justine will sign the report.

11. All forms of advertisements must have prior review and approval by the
a. Lead engagement partner
b. Risk management partner
c. Board of Accountancy
d. Quality Control reviewer

12. Which of the following is not a broad category of safeguards that mitigate or eliminate threats to
independence?
a. Safeguards created by the profession, legislation or regulation.
b. Safeguards created to assure proper training within both the client and attest environment.
c. Safeguards implemented by the attest client.
d. Safeguards implemented by the firm, including policies and procedures to implement
professional and regulatory requirements.

13. Offer of gifts or undue hospitality from a client may create threats to objectivity. In evaluating the
significance of threat created by such offer, the professional accountant should consider the:
Nature of the offer Value of the offer Intent behind the offer
a. Yes Yes Yes
b. Yes No Yes
c. No No No
d. No Yes Yes

14. The Code of Professional Ethics would most likely be violated if an auditor
a. Owns a building and leases floor space to an assurance client.
b. Has an insured account with a brokerage firm audit client.
c. Is engaged by an audit client to identify potential acquisitions.
d. Screens candidates for an audit clients vacant controllership position.

15. An audit independence issue might be raised by the auditors participation in consulting services
engagements. Which of the following statements is most consistent with the professions attitude
toward this issue?
a. Information obtained as a result of a consulting services engagement is confidential to that
specific engagement and should not influence performance of the attest function.
b. The decision as to loss of independence must be made by the client based on the facts of the
particular case.
AT Reviewer Part III Page 3

c. The auditor should not make management decisions for an audit client.
d. The auditor who is asked to review management decisions, is also competent to make these
decisions and can do so without loss of independence.

16. Contingency fee based pricing of accounting services is


a. Always strictly prohibited in public accounting practice.
b. Never restricted in public accounting practice.
c. Prohibited for clients for whom attestation services are prohibited.
d. Considered an act discreditable to the profession.

17. If Revised Code of Ethics for Professional Accountants in the Philippines does not specifically
address a threat to auditors independence, the auditor should
a. Conclude that the threat is not significant unless proven so.
b. Conclude that the threat results in a lack of independence unless it can be shown that no
impairment of independence occurs.
c. Consider the threat from the perspective of a reasonable informed third party who has
knowledge of all the relevant information.
d. Consult the PSA for guidance.

18. In which of the following circumstances would an auditor be considered independent when
performing the audit of a new client for the year ended December 31, 2011?
a. The auditor resigned on January 17, 2012 from the board of directors of the client, prior to
accepting the new audit engagement.
b. The auditor continues to hold an immaterial indirect financial interest in the client.
c. The auditor continues to serve as a trustee for the clients pension plan and has the authority to
make investment decisions.
d. The auditors spouse owns an immaterial amount of shares of ordinary shares in the client.

19. The Code of Ethics for Professional Accountants in the Phils. states that when a financial statement
audit client becomes a listed entity, the length of time the engagement partner or the individual
responsible for the engagement quality control review has served the audit client in that capacity
should be considered in determining when the individual should be rotated. Before rotating off the
engagement, the person is allowed to continue to serve as the engagement partner or as the
individual responsible for the engagement quality review for
a. 3 additional years c. 2 additional years
b. 1 additional year d. 4 additional years

20. Which of the following statements concerning commissions is incorrect?


a. A professional accountant in public practice should not pay a commission to obtain a client nor
should a commission be accepted for referral of a client to a third party.
b. A professional accountant in public practice should not accept a commission for the referral of
the products or services of others.
c. Payment and receipt of referral fees between professional accountants in public practice when
no services are performed by the referring accountant are not regarded as commissions.
d. A professional accountant in public practice may enter into an arrangement for the purchase of
whole or part of an accounting practice requiring payments to individuals formerly engaged in the
practice or payments to their heirs or estates.

21. Which of the following is not an appropriate safeguard against client issues?
a. Understanding the clients business activities
b. Obtaining knowledge of the client, its owners, managers and those responsible for its
governance
c. Preparing an engagement letter to document terms of engagement
d. Securing the clients commitment to improve corporate governance practices or internal controls

22. A client company has not paid its 2011 audit fees. According to the Revised Code of Ethics for
Professional Accountants in the Philippines, for the auditor to be considered independent with
respect to the 2012 audit, the 2011 audit fees must be paid before the
a. 2011 report is issued. c. 2012 report is issued
b. 2012 fieldwork is started. d 2013 fieldwork is started.

23. A CPA in public practice should not


a. Use the term accredited c. Indicate membership in foreign firm.
b. Publish services in billboards d. Maintain website in the Internet

24. Professional accountants press and other media releases undertaken to commemorate anniversaries
in public practice by informing the public of their achievements or contributions towards nation
building are:
a. Considered violation of the rules on advertising
b. Forms of solicitation
c. Not permitted in the Philippines
d. Allowed provided such undertaking is done only once every 5 years
AT Reviewer Part III Page 4

25. Any advertising by professional accountants beyond their name, address, telephone number and
membership in professional organizations has been traditionally considered unethical in the
Accountancy Profession, due to the following reasons, except:
a. Advertising could lead to undue competition between and among practitioners, causing a decline
in quality of service
b. Advertising would encourage a more personal approach to clients
c. The cost of advertising would outweigh any savings that might result from competition
d. Small or new practitioners would be unlikely to have the financial resources to match the
advertising of larger or more established practices

26. Which of the following cannot be mentioned by an auditor in publicizing a book in accounting?
a. Name c. Membership in professional organization
b. Qualifications d. Services that the authors firm provides.

27. Under the Revised Code of Ethics for Professional Accountants in the Philippines, which of the
following may a practicing CPA do in connection with educational seminars?
a. Send announcements to non-clients about his appearance on a dominant program or invite them
to attend.
b. Sponsor a seminar and send them invitations to non-clients.
c. Allow himself to be listed as a tax expert on the seminar announcement.
d. Distribute firm literature on relevant topics being discussed at the seminar to non-clients
attending the seminar.

28. Francis, CPA, performs accounting services for Twin Corporation. Norton wishes to offer shares to
the public and asks Francis to audit the financial statements. Francis refers Norton to Ronald, CPA,
who is more competent in the area of registration statements. Ronald performs the audit of Twin's
financial statements and subsequently thank Francis for the referral by giving Francis a portion of the
audit fee. Francis accepts the fee. Who, if anyone, has violated professional ethics?
a. Only Francis. c. Only Ronald.
b. Both Francis and Ronald. d. Neither Francis nor Ronald

29. A CPAs retention of a clients records to enforce payment of overdue fees would be considered
a. An impairment of professional competency.
b. A discreditable act.
c. Acceptable under professional standards.
d. A violation of commercial law.

30. In which of the following instances would the independence of the CPA not be considered to be
impaired? The CPA has been retained as the auditor of a brokerage firm
a. Which owes the CPA audit fees for more than one year.
b. In which the CPA has a large active margin account.
c. In which the CPA's brother is the controller.
d. Which owes the CPA audit fees for services in the current year and has just filed a petition for
bankruptcy.

Philippine Accountancy Act of 2004 (RA 9298) and its IRR

31. What is the effectivity date of the BOA Resolution No. 88 (Series of 2008) prescribing the rules and
regulations for the accreditation of accounting teachers?
a. June 4, 2008 c. December 31, 2008
b. June 20, 2008 d. June 30, 2008

32. The Certificate of Accreditation issued by the PRC to an accounting teacher shall
a. Be valid for 2 years and renewable every 2 years
b. Be valid initially for 3 years and renewable annually
c. Remain in full force and effect unless revoked, cancelled or withdrawn
d. Be valid for 3 years and renewable every 3 years

33. According to the Transitory Provisions of the BOA Resolution No. 88, any tenured/full time/full load
faculty member who does not meet the accreditation requirements as of the effectivity date of the
Rules and Regulations may be issued a Provisional Accreditation. Choose the correct statement.
I. The Provisional Accreditation will be valid for a period not exceeding 3 years unless earlier
withdrawn, revoked, or cancelled for cause by the BOA
II. The Provisional Accreditation may be issued only once and is not renewable
III. The transitory provision shall also apply to returning teachers who have not been teaching for
the last 5 years
a. I only c. II and III only
b. I and II only d. I, II and III
AT Reviewer Part III Page 5

34. Which of the following statements is correct?


a. Any candidate who fails in two (2) complete CPA board examinations who will no longer be
allowed to take another set of examinations
b. Any candidate who fails in two (2) complete CPA board examinations shall be disqualified from
taking another set of examinations unless he/she submits evidence to the satisfaction of the
Board that he/she enrolled in and completed at least twenty-four (24) units of subjects given in
the licensure examination
c. The examination in which the candidate was conditioned and the removal examination on the
subject in which he/she failed shall be counted as two (2) complete examinations
d. The refresher course should be completed within two (2) years from the preceding examination

Glossary of Terms (December, 2007)

35. The date on which those with recognized authority assert that they have prepared the entitys
complete set of financial statements, including the related notes, and that they have taken
responsibility for them
a. Date of the financial statements c. Date of the auditors report
b. Date of approval of financial statements d. Date the financial statements are issued.

36. The date of the end of the latest period covered by the financial statements, which is normally the
date the most recent balance sheet in the financial statements subject to audit.
a. Date of the financial statements c. Date of the auditors report
b. Date of approval of financial statements d. Date the financial statements are issued.

37. Exists in other information when such information, not related to matters appearing in the audited
financial statements, is incorrectly stated or presented.
a. Material inconsistency c. Material misstatement of fact
b. Material weakness d. Significant deficiency

38. Manual or automated procedures that typically operate at a business process level. These controls
can be preventative or detective in nature and are designed to ensure the integrity of the accounting
records.
a. General IT controls c. Access controls
b. Application controls d. Internal controls

39. A combination of hardware and software that protects a WAN, LAN or PC from unauthorized access
through the Internet and from the introduction of unauthorized or harmful software, data or other
material in electronic form.
a. Cryptography c. Firewall
b. Anti-virus software d. Access control

40. An auditor who is asked to replace an existing auditor.


a. Incoming auditor c. Proposed auditor
b. Successor auditor d. Other auditor

41. A process comprising an ongoing consideration and evaluation of the firms system of quality control.
a. Coordination c. Direction
b. Supervision d. Monitoring

42. A risk that requires special audit consideration.


a. Audit risk c. Business risk
b. Significant risk d. Engagement risk

43 A matter whose outcome depends on future actions or events not under the direct control of the
entity but that may affect the financial statements
a. Uncertainty c. Provision
b. Inconsistency d. Significant risk

44. Subsequent events refer to events that occur after the date of the financial statements and are:
a. Favorable to the entity being audited
b. Unfavorable to the entity being audited
c. Either favorable or unfavorable to the entity being audited
d. Neither favorable nor unfavorable to the entity being audited

45. It is used to categorize a wide variety of financial instruments whose value depends on an underlying
rate or price, such as interest rates, exchange rates, equity prices or commodity prices.
a. Asset/liability management c. Derivatives
b. Commodity d. Hedge
AT Reviewer Part III Page 6

Preface to Philippine Standards on Auditing and Related Services / PSA 120, Framework of
Philippine Standards on Auditing/ PSA 200 Revised and Redrafted Overall Objective of the
Independent Auditor and the Conduct of an Audit in Accordance with PSA/PSRE 2400
Engagements to Review FS/PSAE 3000 Rev Assurance Engagements Other than Audits or
Review of Historical FS/PSRS 4400 Engagements to Perform Agreed-Upon Procedures/PSRS 4410
Engagements to Compile Financial Information

46. Which of the following statements about independent financial statement audit is incorrect?
a. The risk that the auditor will fail to uncover material misstatement is eliminated when the auditor
complies with PSA.
b. The auditors opinion enhances the credibility of the financial statements.
c. The phrase used to express the auditors opinion is present fairly, in all material respects..
d. The term scope of audit refers to audit procedures deemed necessary in the circumstances to
achieve the objective of the audit.

47. Which of the following best describes the benefit of having an annual financial statement audit?
a. Meet the requirements of the government agencies.
b. Provide assurance that errors and fraud, if any exist, will be brought to light.
c. Enable officers and directors to avoid personal responsibility for any misstatements in the
financial statements.
d. Provide assurance to investors and other outsiders that the financial statements are dependable.

48. Which of the following factors are essential to an effective internal auditing organization?
I. Operating responsibility III. Objectivity
II. Organizational status IV. Authority over operations
a. I and II. c. III and IV
b. II and III. d. I and IV.

49. To operate effectively, internal auditor must be independent of


a. The line functions of the organization.
b. The employer-employee relationship which exists for other employees in the organization.
c. The entity.
d. All of the following.

50. The overall objective of internal auditing is to


a. Attest to the efficiency with which resources are employed.
b. Provide assurance that financial data have been accurately recorded.
c. Assist members of the organization in the effective discharge of their responsibilities.
d. Ascertain that controls are cost justified.

51. The work of internal auditors may affect the independent auditors
I. Procedures performed in obtaining an understanding of internal control
II. Procedures performed in assessing the risk of material misstatement
III. Substantive procedures performed in gathering direct evidence
a. I and II only c. II and IIII only.
b. I and III only d. I. II and III.

52. For which of the following judgments may an independent auditor share responsibility with an entitys
internal auditor who is assessed to be both competent and objective?
Assessment of inherent risk Assessment of control risk
a. Yes Yes
b. Yes No
c. No Yes
d. No No

53. During an audit an internal auditor may provide direct assistance to an independent CPA in
Obtaining understanding Performing tests Performing
of internal control of controls substantive tests
a. No No No
b. Yes No No
c. Yes Yes No
d. Yes Yes Yes

54. The primary difference between operational auditing and financial auditing is that in operational
auditing
a. The auditor is not concerned with whether the audited activity is generating information in
compliance with financial accounting standards.
b. The auditor is seeking to help management use resources in the most effective manner possible.
c. The auditor starts with the financial statements of an activity being audited and works backward
to the basic processes involved in producing them.
d. The auditor can use analytical skills and tools that are not necessary in financial auditing.
AT Reviewer Part III Page 7

55. Which of the following is not a distinction between a compilation and a review?
a. The CPA must be independent as a prerequisite to performing a review engagement, but need
not be independent to perform a compilation.
b. In conducting a review, the CPA must obtain an understanding of the client's internal control
system; but this is not necessary for a compilation engagement.
c. Analytical procedures are applied in a review engagement, but are not required in a compilation.
d. A compilation offers no assurance, whereas a review provides limited assurance.

56. Which of the following is not an example of the application of professional skepticism?
a. Designing additional auditing procedures to obtain more reliable evidence in support of a
particular financial statement assertion.
b. Obtaining corroboration of management's explanations through consultation with a specialist.
c. Inquiring of prior year engagement personnel regarding their assessment of management's
honesty and integrity.
d. Using third party confirmations to provide support for management's representations.

57. An auditor has been asked to report on the statement of financial position of Kane Company but not
on the other basic financial statements. The auditor will have access to all information underlying the
basic financial statements. Under these circumstances, the auditor:
a. May accept the engagement because such engagements merely involve limited reporting
objectives.
b. May accept the engagement but should disclaim an opinion because of an inability to apply the
procedures considered necessary.
c. Should refuse the engagement because there is a client-imposed scope limitation.
d. Should refuse the engagement in accordance with PSAs.

58. The objective of a review of interim financial information for a non-issuer is to provide the accountant
with a basis for communicating whether
a. A reasonable basis exists for expressing an updated opinion regarding the financial statements
that were previously audited
b. Material modifications should be made to conform with the applicable financial reporting
framework
c. The financial statements are presented fairly in accordance with standards of interim reporting
d. The financial statements are presented fairly in conformity with the applicable financial reporting
framework

59. A practitioner has been engaged to apply agreed-upon procedures in accordance with Philippine
Standards on Related Services (PSRS) to prospective financial statements. Which of the following
conditions must be met for the practitioner to perform the engagement?
a. The prospective financial statement includes a summary of significant accounting policies.
b. The practitioner takes responsibility for the sufficiency of the agreed-upon procedures.
c. The practitioner and specified parties agreed upon the procedures to be performed by the
practitioner.
d. The practitioner reports on the criteria to be used in the determination of findings.

60. An agreed-upon procedures engagement may involve the accountant in performing certain
procedures concerning
I. Individual items of financial data.
II. A financial statement.
III. A complete set of financial statements.
a. I and II only. c. I and III only.
b. II and III only. d. I, II and III.

61. Accepting an engagement to compile an entitys financial projection most likely would be
inappropriate if the projection is to be included in a(an)
a. Mortgage application for the purpose of expanding the entitys facilities.
b. Offering statement of the entitys initial public offering of ordinary shares.
c. Comprehensive document to be used in negotiating a new labor contract
d. Report to the audit committee that is not sent to the stockholders.

62. Which of the following procedures is an accountant required to perform when reviewing the financial
statements of a nonpublic entity in accordance with Philippine Standards on Review Engagement?
a. Assess control risk.
b. Obtain a management representation letter.
c. Confirm account balances.
d. Perform a physical inventory observation.

63. An accountant agrees to the clients request to change an engagement from a review to a compilation
of financial statements. The compilation report should include
a. No reference to the original engagement.
b. Reference to a departure from GAAS.
c. Scope limitations that may have resulted in the change of engagement.
d. Information about review procedures already performed.
AT Reviewer Part III Page 8

64. Which of the following statements is correct regarding a review of a non-public entitys financial
statements in accordance with Philippine Standards on Review Engagement (PSRE)?
a. The accountant is required to assess the risk of fraud.
b. It is not necessary for the accountant to obtain a management representation letter.
c. An opinion is expressed in the review report.
d. The accountant must be independent to issue the review report.

65. An accountant has been engaged to compile the financial statements of a non-public entity. The
financial statements contain many departures from GAAP because of inadequacies in the accounting
records. The accountant believes that modification of the compilation report is not adequate to
indicate the deficiencies. Under these circumstances, the accountant should
a. Inform management that the engagement can proceed only if distribution of the accountants
report is restricted to internal use.
b. Withdraw from the engagement and provide no further service concerning these financial
statements.
c. Quantify the effects of the departures from GAAP and describe the departures from GAAP in a
special report.
d. Obtain written representations from management that the financial statements will not be used to
obtain credit from financial institutions.

66. Which of the following actions should an accountant take when engaged to compile a companys
financial statements in accordance with Philippine Standards on Related Services (PSRS)?
a. Perform analytical procedures.
b. Express negative assurance on the financial statements.
c. Make management inquiries and examine internal controls.
d. Perform the engagement even though independence is compromised.

67. Which of the following procedures would a CPA most likely perform when reviewing the financial
statements of a non-issuer?
a. Verify that the accounting estimates that could be material to the financial statements have been
developed.
b. Obtain an understanding of the entitys internal control components.
c. Assess the entitys ability to continue as a going concern for a reasonable period of time.
d. Make inquiries about actions taken at the board of directors meetings.

68. Which of the following procedures is ordinarily performed by an accountant during an engagement to
compile the financial statements of a non-issuer?
a. Make inquiries of the employees and senior management regarding transactions with related
parties.
b. Determine whether there is substantial doubt about the entitys ability to continue as a going
concern.
c. Scan the entitys records for the period just after the balance sheet date to identify subsequent
events requiring disclosure.
d. Consider whether the financial statements are free from obvious material mistakes in the
application of accounting principles.

69. Which of the following statements is correct regarding a compilation report on financial statements
issued in accordance with Philippine Standards on Related Services (PSRS)?
a. The report should not be issued if the accountant is not independent from the entity.
b. The report should include a statement indicating that the information is the representation of the
accountant.
c. The report should include a description of other procedures performed during the compilation.
d. The date of the report should be the date of completion of the compilation.

70. Which of the following statements would be appropriate in an accountants report on compiled
financial statements of a non-issuer prepared in accordance with Philippine Standards on Related
Services (PSRS)?
a. We are not aware of any material modifications that should be made to the accompanying
financial statements.
b. A compilation is substantially less in scope than an audit in accordance with generally accepted
auditing standards (GAAS).
c. A compilation is limited to presenting in the form of financial statements information that is a
representation of management.
d. A compilation is performed to obtain reasonable assurance about whether the financial
statements are free from material misstatement.

71. Which of the following statements is correct concerning both an engagement to compile and an
engagement to review a nonpublic entitys financial statements?
a. The accountant should obtain a written management representation letter
b. The accountant must be independent in fact and appearance
c. The accountant expresses no assurance on the financial statements
d. The accountant does not contemplate obtaining an understanding of internal control
AT Reviewer Part III Page 9

72. Which of the following procedures is not usually performed by the accountant during a review
engagement of a nonissuer?
a. Inquiring about actions taken at meetings of the board of directors that may affect the financial
statements
b. Issuing a report stating that the review was performed in accordance with standards established
by the PICPA.
c. Reading the financial statements to consider whether they conform with generally accepted
accounting principles
d. Communicating any material weaknesses discovered during the consideration of internal control

73. Which of the following procedures is usually performed by the accountant in a review engagement of
a nonpublic entity?
a. Sending a letter of inquiry to the entitys lawyer
b. Comparing the financial statements with statements for comparable prior periods
c. Confirming a significant percentage of receivables by direct communication with debtors
d. Communicating significant deficiencies discovered during the study of internal control

74. Which of the following inquiry or analytical procedures ordinarily is performed in an engagement to
review a nonpublic entitys financial statements?
a. Analytical procedures designed to test the accounting records by obtaining corroborating
evidential matter
b. Inquiries concerning the entitys procedures for recording and summarizing transactions
c. Analytical procedures designed to test managements assertions regarding continued existence
d. Inquiries of the entitys attorney concerning contingent liabilities

75. Which of the following should be the first step in reviewing the financial statements of an entity?
a. Comparing the financial statements with statements for comparable prior periods and with
anticipated results.
b. Completing a series of inquiries concerning the entity's procedures for recording, classifying, and
summarizing transactions.
c. Obtaining a general understanding of the entity's organization, its operating characteristics, and
its products or services.
d. Applying analytical procedures designed to identify relationships and individual items that
appear to be unusual.

76. When an accountant examines projected financial statements, the accountants report should include
a separate paragraph that
a. Describes the limitations on the usefulness of the presentation
b. Provides an explanation of the differences between an examination and an audit
c. States that the accountant is responsible for events and circumstances up to one year after the
reports date
d. Disclaims an opinion on whether the assumptions provide a reasonable basis for the projection

77. Which of the following professional services is considered an attest engagement?


a. A consulting service engagement to provide computer advice to a client.
b. An engagement to report on compliance with statutory requirements.
c. An income tax engagement to prepare national tax returns.
d. The compilation of financial statements from a clients accounting records.

78. Philippine Standards on Related Services (PSRSs) require an accountant to report when the
accountant has
a. Typed client-prepared financial statements, without modification, as an accommodation to the
client.
b. Provided a client with a financial statement format that does not include peso amounts, to be
used by the client in preparing financial statements.
c. Proposed correcting journal entries to be recorded by the client that change client-prepared
financial statements.
d. Prepared, through the use of computer software, financial statements to be used by third parties.

79. An accountant is required to comply with the provisions of Philippine Standards on Related Services
when
I. Reproducing client-prepared financial statements, without modification, as an accommodation to
a client
II. Preparing standard monthly journal entries for depreciation and expiration of prepaid expenses
a. I only c. Both I and II
b. II only d. Neither I nor II
AT Reviewer Part III Page 10

80. How does an accountant make the following representations when issuing the standard report for the
compilation of a nonpublic entitys financial statements?
The financial statements The accountant has compiled the
have not been audited financial statements
a. Implicitly Implicitly
b. Explicitly Explicitly
c. Implicitly Explicitly
d. Explicitly Implicitly

81. An accountant may compile a nonpublic entitys financial statements intended for third-party use that
omit all of the disclosures required by PAS only if the omission is
I. Clearly indicated in the accountants report
II. Not undertaken with the intention of misleading the financial statement users
a. I only c. Both I and II
b. II only d. Either I or II

82. Which of the following representations does an accountant make implicitly when issuing the standard
report for the compilation of a nonpublic entitys financial statements?
a. The accountant is independent with respect to the entity
b. The financial statements have not been audited
c. A compilation consists principally of inquiries and analytical procedures
d. The accountant does not express any assurance on the financial statements

83. When an accountant compiles a financial forecast, the accountant's report should include a(an):
a. Explanation of the differences between a financial forecast and a financial projection.
b. Caveat that the prospective results of the financial forecast may not be achieved.
c. Statement that the accountant's responsibility to update the report is limited to one year.
d. Disclaimer of opinion on the reliability of the entity's internal controls.

84 Which of the following is a professional engagement that a CPA may perform to provide assurance on a
system's reliability?
a. MAS AssurAbility. c. MAS AttestSure
b. CPA WebMaster. d. CPA SysTrust

85. An accountant's standard report on a compilation of a projection should not include a statement that:
a. There will usually be differences between the forecasted and actual results.
b. The hypothetical assumptions used in the projection are reasonable in the circumstances.
c. The accountant has no responsibility to update the report for future events and circumstances.
d. The compilation of a projection is limited in scope.

86. A practitioner's report on agreed-upon procedures that is in the form of procedures and findings should
contain:
a. Negative assurance that the procedures did not necessarily disclose all significant deficiencies in
internal control.
b. An acknowledgment of the practitioner's responsibility for the sufficiency of the procedures.
c. A statement of restrictions on the use of the report.
d. A disclaimer of opinion on the entity's financial statements.

87. In reviewing the financial statements of a nonpublic entity, an accountant is required to modify the
standard review report for which of the following matters?
Inability to Assess the Risk of Material Discovery of Significant Deficiencies
Misstatement Due to Fraud in the Design of the Internal Control
a. Yes Yes
b. Yes No
c. No Yes
d. No No

88. Each page of a nonpublic entitys financial statements reviewed by an accountant should include the
following reference:
a. See Accountants Review Report.
b. Reviewed. No Accountants Assurance Expressed.
c. See Accompanying Accountants Notes.
d. Reviewed. No Material Modifications Required.

89. An accountant began an audit of the financial statements of a nonpublic entity and was asked to
change the engagement to a review because of a restriction on the scope of the audit. If there is
reasonable justification for the change, the review report should include reference to the
Original Engagement Scope Limitation that caused the
That was Agreed to Changed Engagement
a. Yes Yes
b. Yes No
AT Reviewer Part III Page 11

c. No Yes
d. No No

90. Ronald, CPA, accepted an engagement to audit the financial statements of Queen Resources, a
nonpublic entity. Before the completion of the audit, Queen Resources requested Ronald to change
the engagement to a compilation of financial statements. Before Ronald agrees to change the
engagement, Ronald is required to consider the
Additional Audit Effort Necessary Reason Given for
to Complete the Audit Queens Request
a. No No
b. Yes Yes
c. Yes No
d. No Yes

91. An auditors report is designated a special report when it is issued in connection with
a. Interim financial information of a publicly held company that is subject to a limited review
b. Compliance with aspects of regulatory requirements related to audited financial statements
c. Application of accounting principles to specified transactions
d. Limited use prospective financial statements such as financial projection

92. An auditors report issued in connection with which of the following is generally not considered to be
a special report?
a. Compliance with aspects of contractual agreements unrelated to audited financial statements
b. Specified elements, accounts, or items of a financial statement presented in a document
c. Financial statements prepared in accordance with an entitys income tax basis
d. Financial information presented in a prescribed schedule that requires a prescribed form of
auditors report
93. Financial information is presented in a printed form that prescribes the wording of the independent
auditors report. The form is not acceptable to the auditor because the form calls for statements that
are inconsistent with the auditors responsibility. Under these circumstances, the auditor most likely
would
a. Withdraw from the engagement
b. Reword the form or attach a separate report
c. Express a qualified opinion with an explanation
d. Limit use of the report to the party who designed the form

94. An accountant has compiled the financial statements of a nonpublic entity in accordance with
Philippine Standards on Related Services (PSRSs). Do the PSRSs require that the compilation
report be printed on the accountants letterhead and that the report be manually signed by the
accountant?
Printed in the Manually Signed
Accountants Letterhead by the Accountant
a. Yes Yes
b. Yes No
c. No Yes
d. No No

95. When engaged to compile the financial statements of a nonpublic entity, an accountant is required to
possess a level of knowledge of the entitys accounting principles and practices. This requirement
most likely will include obtaining a general understanding of the
a. Stated qualifications of the entitys accounting personnel
b. Design of the entitys internal controls placed in operation
c. Risk factors relating to misstatements arising from illegal acts
d. Internal control awareness of the entitys senior management

96. Which of the following inquiry or analytical procedures ordinarily is performed in an engagement to
review a nonpublic entitys financial statements?
a. Analytical procedures designed to test the accounting records by obtaining corroborating
evidential matter.
b. Inquiries concerning the entitys procedures for recording and summarizing transactions.
c. Analytical procedures designed to test managements assertions regarding continued existence.
d. Inquiries of the entitys attorney concerning contingent liabilities.
.
97. Which of the following presents what the effects on historical financial data might have been if a
consummated transaction had occurred at an earlier date?
a. Prospective financial statements c. Interim financial information
b. Pro forma financial information d. A financial projection

98. A practitioner may report on an examination of pro forma financial information if the related hi storical
financial statements have been
a. Audited c. Audited, reviewed or compiled.
b. Audited or reviewed d. Reviewed or compiled.
AT Reviewer Part III Page 12

99. A financial forecast consists of prospective financial statements that present an entitys expected
financial position, results of operations, and cash flows. A forecast
a. Is based on the most conservative estimates.
b. Presents estimates given one or more hypothetical assumptions.
c. Unlike a projection, may contain a range.
d. Is based on assumptions reflecting conditions expected to exist and courses of action expected
to be taken.

100. An examination of a financial forecast is a professional service that involves


a. Compiling or assembling a financial forecast that is based on managements assumptions
b. Restricting the use of the practitioners report to management and the board of directors
c. Assuming responsibility to update management on key events for one year after the reports date
d. Evaluating the preparation of a financial forecast and the support underlying managements
assumptions

101. Given one or more hypothetical assumptions, a responsible party may prepare, to the best of its
knowledge and belief, an entitys expected financial position, results of operations, and cash flows.
Such prospective financial statements are known as
a. Pro forma financial statements. c. Partial presentations
b. Financial projections. d. Financial forecasts.

102. Which of the following is a prospective financial statement for general use upon which a practitioner
may appropriately report?
a. Financial projection. c. Pro forma financial
statement
b. Partial presentation. d. Financial forecast.

PSA 210 (Redrafted), Agreeing the Terms of Audit Engagements

103. Which of the following circumstances would permit an independent auditor to accept an engagement
after the close of the fiscal year?
a. Issuance of a disclaimer of opinion as a result of inability to conduct certain tests required by
generally accepted auditing standards due to the timing of acceptance of the engagement
b. Assessment of control risk below the maximum level
c. Receipt of an assertion from the preceding auditor that the entity will be able to continue as a
going concern
d. Remedy of limitations resulting from accepting the engagement after the close of the end of the
year, such as those relating to the existence of physical inventory

104. Which of the following is not normally performed in the pre-engagement phase?
a. Assessing control risk.
b. Deciding whether to accept or reject an audit engagement.
c. Inquiring from predecessor auditor.
d. Preparing an engagement letter.

105. An incoming auditor plans to consult the predecessor auditor and review certain portion of the
predecessor auditors working papers. This procedure is acceptable of
a. The predecessor auditor and the client consent.
b. The client consents.
c. The incoming auditor consents.
d. The predecessor auditor consents.

106. Engagement letters


a. Are mandated for audit engagements.
b. Are recommended for all professional engagements.
c. Are signed by members of the audit committee.
d. Include an analysis of the results of the audit.

PSA 230`Redrafted, Audit Documentation

107 Per PSA, audit documentation (working papers)


a. Must be completed within 60 days after the auditors fieldwork has completed.
b. Includes peer review reports for the audit firm.
c. Must be sent to the SEC for its review.
d. Is to be retained for five years from the date of issue of the report.

108 Audit documentation (working papers)


a. Facilitates the planning, performance, and supervision of the engagement.
b. Becomes the property of the client once the opinion is issued.
c. Is required for public company audits and recommended, but not required, for non-public
company audits.
d. Does not need to contain evidence that an understanding of internal control was obtained.
AT Reviewer Part III Page 13

109. Which of the following is required documentation in an audit in accordance with PSA?
a. A flowchart or narrative of the information system relevant to financial reporting describing the
recording and classification of transactions for financial reporting.
b. An audit plan setting forth in detail the procedures necessary to accomplish the engagement's
objectives.
c. A planning memorandum establishing the timing of the audit procedures and coordinating the
assistance of entity personnel.
d. An internal control questionnaire identifying controls that assure specific objectives will be
achieved.

110. If working papers are to have the characteristics that will ensure that they achieve their primary
purposes, which of the following is the most important?
a. Working papers must be of standard format and standard content.
b. Working papers must be properly indexed and cross- referenced to the draft audit report.
c. Working papers must provide sufficient, competent, and useful information to support the audit
report.
d. Working papers must be arranged in logical order following the audit program sequence.

111. Which of the following does not describe one of the functions of audit working papers?
a. Facilitates third-party reviews.
b. Aids in the planning, performance, and review of audits.
c. Provides the principal evidential support for the auditor's report.
d. Aids in the professional development of the operating staff.

112. Audit documentation should be prepared in enough detail so that:


a. An experienced auditor who has worked with the client in the past can understand the
procedures performed and the evidence obtained.
b. A reader of the financial statements who has no previous connection with the audit can
understand the procedures performed and the evidence obtained.
c. A reader of the financial statements who has a background in financial analysis can understand
the procedures performed and the evidence obtained.
d. An experienced auditor who has no previous connection with the audit can understand the
procedures performed and the evidence obtained.

113 Which of the following is a true statement regarding documentation requirements for analytical
procedures?
a. When an analytical procedure is used as the principal substantive test of a significant financial
statement assertion, the auditor is required to document the reasons analytical procedures were
performed instead of tests of details.
b. When an analytical procedure is used as the principal substantive test of a significant financial
statement assertion, the auditor is required to document his or her expectation and
management's concurrence with that expectation.
c. When an analytical procedure is used during the overall review stage of the audit, the auditor is
required to document the auditor's expectation and any additional procedures performed to
investigate significant unexplained differences.
d. When an analytical procedure is used as the principal substantive test of a significant financial
statement assertion, the auditor is required to document both the auditor's expectation and the
factors considered in developing that expectation.

PSA 240 Redrafted, The Auditors Responsibilities Relating to Fraud in an Audit of Financial
Statements

114. Which of the following statements best describes the auditors responsibility regarding the detection
of material errors and fraud?
a. The auditor is responsible for the failure to detect material errors and fraud only when such
failure results from the non-application of generally accepted accounting principles
b. Extended auditing procedures are required to detect material errors and fraud if the audit
indicates that they may exist
c. The auditor is responsible for the failure to detect material errors and fraud only when the auditor
fails to confirm receivables or observe inventories
d. Extended auditing procedures are required to detect unrecorded transactions even if there is no
evidence that material errors and fraud may exist

115. Which of the following statements best describes the auditor's responsibility regarding the detection
of material fraud?
a. Because of the inherent limitations of an audit, the auditor is not responsible for the failure to
detect material fraud.
b. The auditor is responsible for the failure to detect material fraud when such failure results from
nonperformance of audit procedures specifically described in the engagement letter.
c. The auditor should design audit programs that will provide reasonable assurance that material
errors and fraud will be detected in the ordinary course of the examination.
d. The auditor is responsible for the failure to detect material fraud when the auditor's evaluation of
internal control procedures indicates that they are ineffective.
AT Reviewer Part III Page 14

116. Early in an audit, the auditor discovered several fraud risk factors. Which of the following is least
likely response of the auditor?
a. Substantive test procedures are moved away from the end of the clients accounting period so
that differences can be more easily resolved.
b. The auditor should attempt to gather more audit evidence through physical inspection,
c. The auditor should attempt to incorporate in the audit more elements of unpredictability.
d. Analytical procedures should be still applied but a more disaggregated level.

117. As a result of analytical procedures, the independent auditor determines that the gross profit
percentage has declined from 30 percent in the preceding year to 20 percent in the current year.
The auditor should
a. Express a qualified opinion due to inability of the client to continue as a going concern.
b. Evaluate managements performance in causing this decline.
c. Require note disclosure.
d. Consider the possibility of a misstatement in the financial statements.

118. The management responsibility to detect and prevent fraud and error is accomplished by
a. Having an annual audit of financial statements.
b. Establishing a control environment and implementing adequate internal control policies and
procedures.
c. Signing the management representation letter.
d. Implementing adequate quality control system.

119. Which of the following relatively small misstatements most likely could have a material effect on an
entitys financial statements?
a. An illegal payment to a foreign official that was not recorded
b. A piece of obsolete office equipment that was not retired
c. A petty cash fund disbursement that was not properly authorized
d. An uncollectible account receivable that was not written off

105. Which of the following situations represents a risk factor that relates to misstatements arising from
misappropriation of assets?
a. A high turnover of senior management
b. A lack of independent checks
c. A strained relationship between management and the predecessor auditor
An inability to generate cash flow from operations

106 Which of the following procedures would an auditor most likely perform during the overall review
stage of an audit of an entitys financial statements?
a. Obtain assurance from the entitys attorney that all material litigation has been disclosed in the
financial statements.
b. Verify the clerical accuracy of the entitys proof of cash and its bank cutoff statement.
c. Determine whether inadequate provisions for the safeguarding of assets have been corrected.
d. Consider whether the results of audit procedures affect the assessment of the risk of material
misstatement due to fraud.

107. Which of the following statements is correct concerning an auditors responsibility to report fraud?
a. The auditor is required to communicate to the clients audit committee all minor fraudulent acts
perpetrated by low-level employees, even if the amounts involved are inconsequential.
b. The disclosure of material management fraud to principal stockholders is required when both
senior management and the board of directors fail to acknowledge the fraudulent activities.
c. Fraudulent activities involving senior management of which the auditor becomes aware should
be reported directly to the SEC.
d. The disclosure of fraudulent activities to parties other than the clients senior management and
its audit committee is not ordinarily part of the auditors responsibility.

108. Which of the following circumstances most likely would cause an auditor to suspect that there are
material misstatements in an entitys financial statements?
a. The entitys management places no emphasis on meeting publicized earnings projections
b. Significant differences between the physical inventory count and the accounting records are not
investigated
c. Monthly bank reconciliations ordinarily include several large outstanding checks
d. Cash transactions are electronically processed and recorded, leaving no paper audit trail

109. If the business environment is experiencing a recession, the auditor most likely would focus
increased attention on which of the following accounts?
a. Purchase returns and allowances.
b. Allowance for doubtful accounts.
c. Ordinary shares.
d. Noncontrolling interest of a subsidiary purchased during the year.
AT Reviewer Part III Page 15

110. Which of the following circumstances would an auditor most likely consider a risk factor relating to
misstatements arising from fraudulent financial reporting?
a. Several members of management have recently purchased additional shares of the entitys
stock.
b. Several members of the board of directors have recently sold shares of the entitys stock.
c. The entity distributes financial forecasts to financial analysts that predict conservative operating
results.
d. Management is interested in maintaining the entitys earnings trend by using aggressive
accounting practices.

111. When assessing the risk of material misstatement, an auditor is required to document
I. The basis for the assessment
II. Significant risks identified and the related controls that were evaluated
a. I only
b. II only
c. Both I and II
d. Neither I nor II

112. Which of the following characteristics most likely would heighten an auditors concern about the risk
of material misstatement arising from fraudulent financial reporting?
a. There is a lack of interest by management in maintaining an earnings trend.
b. Computer hardware is usually sold at a loss before being fully depreciated.
c. Management had frequent disputes with the auditor on accounting matters.
d. Monthly bank reconciliations usually include several large checks outstanding.

113. Which of the following factors or conditions is an auditor least likely to plan an audit to discover?
a. Financial pressures affecting employees.
b. High turnover of senior management.
c. Inadequate monitoring of significant controls.
d. Inability to generate positive cash flows from operations.

114. At which stage (s) of the audit may fraud risk factors be identified?
Obtaining an understanding of
Planning Internal control Conducting fieldwork

a. Yes Yes Yes


b. Yes Yes No
c. Yes No No
d. No Yes Yes

PSA 250 Redrafted , Consideration of Laws and Regulations in Audit of Financial Statements

115. Which of the following information discovered during an audit most likely would raise a question
concerning possible noncompliance?
a. Related party transactions, although properly disclosed, were pervasive during the year.
b. The entity prepared several large checks payable to cash during the year.
c. Material internal control weaknesses previously reported to management were not corrected.
d. The entity was a campaign contributor to several local political candidates during the year.

116. Which of the following procedures would least likely result in the discovery of possible
noncompliance?
a. Reading the minutes of the board of director meetings.
b. Making inquiries of the clients management.
c. Performing tests of details of transactions.
d. Reviewing an internal control questionnaire.

117. When an auditor becomes aware of a possible illegal act by a client, the auditor should obtain an
understanding of the nature of the act to
a. Evaluate the effect on the financial statements.
b. Determine the reliability of managements representations.
c. Consider whether other similar acts may have occurred.
d. Recommend remedial actions to the audit committee.

118. If information comes to an auditors attention that implies the existence of possible noncompliance
that could have a material, but indirect effect on the financial statements, the auditor should next
a. Apply audit procedures specifically directed to ascertaining whether an illegal act has occurred.
b. Seek the advice of an informed expert qualified to practice law as to possible contingent
liabilities.
c. Discuss the evidence with the clients audit committee, or others with equivalent authority and
responsibility.
d. Report the matter to an appropriate level of management at least one level Above those
involved.
AT Reviewer Part III Page 16

PSA 260 Revised and Redrafted ,Communication with Those Charged with Governance

119. Which of the following matters would an auditor most likely communicate to an entitys audit
committee?
a. A list of negative trends that may lead to working capital deficiencies and adverse financial
ratios.
b. The level of responsibility assumed by management for the preparation of the financial
statements.
c. Difficulties encountered in achieving a satisfactory response rate from the entitys customers in
confirming accounts receivables.
d. The effects of significant accounting policies adopted by management in emerging areas for
which there is no authoritative guidance.

120. An auditors communication with the audit committee is required to include the
a. Basis for the auditors preliminary judgment About materiality.
b. Justification for the auditors selection of sampling methods.
c. Discussion of disagreements with management about matters that significantly impact the
entitys financial statements.
d. Assessment of the quality of the entitys earnings as compared to the previous year.

121. An auditor would least likely initiate a discussion with a clients audit committee concerning
a. The methods used to account for significant unusual transactions.
b. The maximum peso amount of misstatements that could exist without causing the financial
statements to be materially misstated.
c. Indications of fraud and illegal acts committed by a corporate officer that were discovered by the
auditor.
d. Disagreements with management as to accounting principles that were resolved during the
current years audit.

122. In identifying matters for communication with an entitys audit committee, an auditor most likely would
ask management whether
a. The turnover in the accounting department was unusually high.
b. It consulted with another CPA firm about accounting matters.
c. There were any subsequent events of which the auditor was unaware.
d. It agreed with the auditors assessed level of control risk.

123. Which of the following matters is an auditor not required to communicate to an entitys audit
committee?
a. Significant adjustments arising from the audit that were recorded by management.
b. The basis for the auditors conclusions about the reasonableness of managements sensitive
accounting estimates.
c. The level of responsibility assumed by the auditor under generally accepted auditing standards.
d. The degree of reliance the auditor placed on the management representation letter.

PSA 300 Redrafted, Planning an Audit of Financial Statements

124. Which of the following procedures would an auditor most likely include in the initial planning of a
financial statement audit?
a. Obtaining a written representation letter from the clients management.
b. Examining documents to detect illegal acts having a material effect on the financial statements.
c. Considering whether the clients accounting estimates are reasonable in the circumstances.
d. Determining the extent of involvement of the clients internal auditors.

125 When assessing the internal auditors' competence, the independent CPA should obtain information
about the:
a. Organizational level to which the internal auditors report.
b. Educational background and professional certification of the internal auditors.
c. Policies prohibiting the internal auditors from auditing areas where relatives are employed.
d. Internal auditors' access to records and information that is considered sensitive.

126. Of the following procedures, which is not considered part of obtaining an understanding of the
clients environment?
a. Examining trade publications to gain a better understanding of the client's industry.
b. Confirming customer accounts receivable for existence and valuation.
c. Touring the client's manufacturing and warehousing facilities to gain a clearer understanding of
operations.
d. Studying the internal controls over cash receipts and disbursements.
AT Reviewer Part III Page 17

127. An initial (first-time) audit requires more audit time to complete than a recurring audit. One of the
reasons for this is that
a. New auditors are usually assigned to an initial audit.
b. Predecessor auditors need to be consulted.
c. The client's business, industry, and internal control are unfamiliar to the auditor and need to be
carefully studied.
d. A larger proportion of customer accounts receivable need to be confirmed on an initial audit.

128. Prior to commencing fieldwork, an auditor usually discusses the general audit strategy with the
clients management. Which of the following details do management and the auditor usually agree
upon at this time?
a. The specific matters to be included in the communication with the audit committee.
b. The minimum amount of misstatements that may be considered to be significant deficiencies and
material weaknesses.
c. The schedules and analyses that the clients staff should prepare.
d. The effects that inadequate controls may have over the safeguarding of assets.

129. The in-charge auditor most likely would have a supervisory responsibility to explain to the staff
assistants
a. That immaterial fraud is not to be reported to the clients audit committee.
b. How the results of various auditing procedures performed by the assistants should be evaluated.
c. Why certain documents are being transferred from the current file to the permanent file.
d. What benefits may be attained by the assistants adherence to established time budgets.

130. The senior auditor responsible for coordinating the fieldwork usually schedules a pre-audit
conference with the audit team primarily to
a. Give guidance to the staff regarding both technical and personnel aspects of the audit.
b. Provide an opportunity to document staff disagreements regarding technical issues.
c. Establish the need for using the work of experts and internal auditors.
d. Discuss staff suggestions concerning the establishment and maintenance of time budgets.

131. Prior to commencing fieldwork, an auditor usually discusses the general audit strategy with the
clients management. Which of the following matters does the auditor and management agree upon
at this time?
a. The appropriateness of the entitys plans for dealing with adverse economic conditions.
b. The determination of the fraud risk factors that exist within the clients operations.
c. The control weaknesses to be included in the communication with the audit committee.
d. The coordination of the assistance of the clients personnel in data preparation.

132. If the business environment is experiencing a recession, the auditor most likely would focus
increased attention on which of the following accounts?
a. Purchase returns and allowances.
b. Allowance for doubtful accounts.
c. Common stock.
d. Noncontrolling interest of a subsidiary purchased during the year.

133. Which of the following factors would a CPA ordinarily consider in the planning stage of an audit
engagement?
I. Financial statement accounts likely to contain a misstatement
II. Conditions that require extension of audit tests
a. I only.
b. II only.
c. Both I and II.
d. Neither I nor II.

134. 1st In an entity with few, but large accounts receivable, the accounts individually are more important
and the possibility of material errors is greater than in another entity that has greater number of small
accounts aggregating the same total
2nd In industrial and merchandising enterprises, inventories are usually of greater importance to
both financial position and results of operations and accordingly may require relatively more
attention by the auditor than would inventories of a public utility company
a. 1st statement is true, 2nd statement is false
b. 1st statement is false, 2nd statement is true
c. Both statements are true
d. Both statements are false
AT Reviewer Part III Page 18

PSA 315 Redrafted, Identifying and Assessing the Risks of Material Misstatements Through
Understanding the Entity and Its Environment/PSA 320, Materiality in Planning and Performing
an Audit/PSA 330 Redrafted, The Auditors Responses to Assessed Risks

135. Risk in auditing means that the auditor accepts some level of uncertainty in performing the audit
function An effective audit
a. Design audit procedures to achieve the desired level of audit risk.
b. Take any means available to reduce the risk to the lowest possible level.
c. Set the risk level between 5% to 10%.
d. Perform audit procedures first and quantitatively set the risk level before forming an opinion
and writing the report.

135. The concept of materiality would be least important to an auditor when considering the
a. Decision whether to use positive or negative confirmation of accounts receivable.
b. Adequacy of disclosure of a clients illegal act.
c. Discovery of weaknesses in a clients internal control.
d. Effects of a direct financial interest in the client upon the CPAs independence.

136. Which of the following statements best describes why an auditor makes a preliminary estimate of
materiality?
a. An estimate is required by PSAs.
b. The estimate provides a basis for evaluating likely misstatements.
c. The estimate helps the auditor plan the appropriate evidence to accumulate.
d. Estimating materiality early helps the auditor avoid legal liability.

136. Under which of the following conditions would you consider lowering individual item materiality
thresholds.
a. Study of the business and industry, together with the application of analytical procedures,
reveals that the client has enjoyed a surge in sales and gross profit during an industry downturn.
b. Application of analytical procedures shows that the client's gross profit rate is significantly below
last year and also is materially lower than the industry average.
c. Study of internal controls within the revenue cycle reveal material weaknesses.
d. Study of internal controls within the payroll cycle confirm the auditor's belief that few errors have
occurred.

137. Which of the following would be least likely to be considered an objective of the internal control
structure?
a. Checking the accuracy and reliability of accounting data.
b. Detecting management fraud.
c. Encouraging adherence to managerial policies.
d. Safeguarding assets.

137. When assessing the risk of material misstatement, an auditor is required to document
I. The basis for the assessment
II. Significant risks identified and the related controls that were evaluated
a. I only. c. Both I and II
b. II only. d. Neither I nor II

137. While performing an audit, Franz, CPA decides to restrict the risk of material misstatement to 3%.
What must the acceptable level of detection risk be, if inherent risk is 25% and control risk is 40%?
a. 0.3%. c. 30.0%.
b. 12.0% d. 33.3%.

138. Which of the following comes closest to outlining the auditors responsibility for internal control on all
financial statement audits?
a. An understanding of the control environment and the accounting system is necessary, an
understanding of the control structures is necessary for areas in which the auditor is performing
tests of controls.
b. The auditor must obtain an understanding of each of the five internal control elements sufficient
to plan the audit.
c. When tests of controls have been performed, control risk must be assessed at a level less than
the minimum.
d. An understanding of the control environment is necessary, but not of the accounting system or
control procedures unless control risk is to be assessed at a level less than the maximum.

138. Regardless of the assessed level of control risk, an auditor would perform some
a. Tests of controls to determine the effectiveness of internal control policies.
b. Analytical procedures to verify the design of internal control procedures.
c. Substantive tests to restrict detection risk for significant transaction classes.
d. Dual-purpose tests to evaluate both the risk of monetary misstatement and preliminary control
risk.
AT Reviewer Part III Page 19

139. Which of the following is necessary in a financial statement audit?


a. An understanding of internal control relevant to each of an entity's operating units.
b. An understanding of internal control relevant to each of an entity's business functions.
c. An understanding of internal control relevant to an entity's financial reporting objective.
d. An understanding of internal control relevant to an entity's compliance objective.

140. Which of the following controls is least likely to be relevant to a financial statement audit?
a. Procedures that prevent the excess use of materials in production.
b. Policies that relate to compliance with income tax regulations.
c. Use of computer passwords to limit access to data files.
d. Generation of production statistics used to evaluate variances.

141. The auditor should obtain sufficient knowledge of the client's information and communication system
relevant to financial reporting to understand all of the following, except:
a. Classes of transactions in the entity's operations that are significant to the financial statements,
and how those transactions are processed, from initiation to inclusion in the financial
statements.
b. The financial reporting process, including development of significant accounting estimates and
inclusion of appropriate disclosures.
c. The means the entity uses to communicate roles, responsibilities, and significant matters
relating to financial reporting.
d. Control activities related to each account balance, transaction class, and disclosure component
in the financial statements or to every assertion relevant to them

142. For a nonissuer, a control deficiency would be considered a significant deficiency when the
likelihood and magnitude of potential financial statement misstatements are:
Likelihood Magnitude
a. More than remote Material
b. Probable Material
c. More than remote More than inconsequential
d. Probable More than inconsequential

143. For a nonissuer, a control deficiency would be considered a material weakness when the
likelihood and magnitude of potential financial statement misstatements are:
Likelihood Magnitude
a. More than remote Material
b. Probable Material
c. More than remote More than inconsequential
d. Probable More than inconsequential

144. After obtaining an understanding of the entity and its environment and assessing the risk of material
misstatement, an auditor decided to perform tests of controls. The auditor most likely decided that
a. It would be efficient to perform tests of controls that would result in a reduction in planned
substantive tests.
b. Additional evidence to support a further reduction in control risk is not available.
c. An increase in the assessed level of control risk is justified for certain financial statement
assertions.
d. There were many internal control weaknesses that could allow errors to enter the accounting
system.

145. Regardless of how the allocation of the preliminary judgment about materiality was done when the
audit is completed, the auditor must be confident that the combined errors in all accounts are
a. Less than the preliminary judgment.
b. Equal to the preliminary judgment.
c. More than the preliminary judgment.
d. Less than or equal to the preliminary judgment

146. Which of the following statements is not correct?


a. Materiality is relative rather than an absolute concept.
b. Normally, the most important base used as the criterion for deciding materiality is Net Income.
c. Qualitative factors as well as quantitative factors affect materiality..
d. Given equal peso amounts, irregularities are usually considered more important than errors.

PSA 500 Redrafted Audit Evidence, PSA 501 Audit Evidence Specific Considerations for
Selected Items, PSA 505 Revised and Redrafted External Confirmations, PSA 520 Redrafted
Analytical Procedures, PSA 540 Revised and Redrafted Auditing Accounting Estimates including
Fair Value Accounting Estimates and Related Disclosures, PSA 550 Revised and Redrafted
Related Parties, PSA 560 Redrafted Subsequent Events, PSA 570 Redrafted Going Concern,
PSA 580 Revised and Redrafted Written Representations
AT Reviewer Part III Page 20

147. The general audit objectives of validity and completeness emphasize opposite audit concerns
a. Validity deals with potential overstatement and completeness deals with understatement.
b. Validity deals with potential understatement and completeness deals with overstatement.
c. Validity and completeness may each deal with overstatements or understatements, but not in the
same transaction.
d. Validity always deals with overstatements but completeness may deal with either over or
understatements.

148. Which of the following presumptions is correct about the reliability of audit evidence?
a. Information obtained indirectly from outside sources is the most reliable evidential matter.
b. To be reliable, audit evidence should be conclusive rather than persuasive.
c. Reliability of audit evidence refers to the amount corroborative evidence obtained.
d. An effective internal control structure provides more assurance about the reliability of audit
evidence.

149. Which of the following is not an example of analytical evidence?


a. Compared inventory turnover by major class with the prior year on a monthly and quarterly basis.
b. Compared gross profit percentages by major product classes with the prior year.
c. Examined invoices for plant asset additions to determine whether the client had erroneously
recorded ordinary repairs as plant assets.
d. Examined monthly performance reports and investigated significant variations from budgeted
amounts.

150. An auditor test counted a batch of inventory. This is an example of what kind of evidence?
a. Analytical. c. Documentary.
b. Physical. d. Hearsay.

151. An auditor wants to develop an audit test to evaluate the reasonableness of the quantity of scrap
material resulting from a certain production process compared to industry standards. Which
would be the most competent type of evidence available to satisfy this objective?
a. Documentary. c. Hearsay.
b. Physical. d. Analytical.

152. A letter to the auditor in response to an inquiry is an example of


a. Physical evidence. c. Documentary evidence
b. Confirmation evidence. d. Analytical evidence.

153. Which of the following would be least likely to be comparable between similar corporations in
the same industry line of business?
a. Earnings per share. c. Accounts receivable turnover/
b. Return on total assets before interest and taxes. d. Operating cycle

153. Transaction cycles begin and end


a. At the beginning and end of the fiscal period. c. At January 1 and December 31.
b. At the balance sheet date. d. At the origin and final disposition of the
entity

154. An objective of a walk-through is to


a. Verify that the structure has been placed in operation.
b. Replace tests of controls.
c. Evaluate the major strengths and weaknesses in the clients structure.
d. Identify weaknesses to be communicated to management in the written representation letter.

154. Generally, what source of evidence should most impact audit conclusions?
a. External b. Inquiry.
c. Oral. d. Informal.

155. Most of the independent auditor's work in formulating an opinion on the financial statements
consists of
a. Studying and evaluating internal control.
b. Obtaining and examining evidential matter.
c. Examining cash transactions.
d. Comparing recorded accountability with assets.
AT Reviewer Part III Page 21

156. Choose the best illustration of objective audit evidence from the following
a. The paid invoice file containing invoices matched with receiving reports and purchase orders.
a. Management's assertion that payment procedures require matching of invoice with
receiving report and purchase order.
c. Clerical staff assurances that management policy regarding payment of invoices--matching of
invoice with receiving report and purchase order--is always followed.
d. The treasurer's statement of not remembering any exceptions in which an invoice was
submitted for payment that was not accompanied by a matching receiving report and purchase
order.

157. Which of the following is a "Type I" subsequent event?


a. The client's Long Island warehouse was destroyed by fire two weeks following the balance
sheet date. The warehouse and its contents were uninsured and represented 15% of the
client's total assets.
b. As the result of an uninsured flood loss, one of the client's major customers declared
bankruptcy. The client doesn't expect to recover more than 5% of the outstanding receivable
which accounts for 30% of total accounts receivable. The flood and bankruptcy declaration
both occurred after the balance date but before the release of the audit report. No additional
provision for loss had been made as of year end.
c. Three weeks after the balance sheet date, a major strike was called by the labor union
representing 80% of the client's work force.
d. After the balance sheet date, but prior to release of the audit report, a product liability judgment
against the client was rendered by a judge. The judgment assessed damages and fines totaling
30% of audited net income. The events giving rise to the judgment occurred prior to the
balance sheet date. The client does not plan to appeal the decision.

158. Which of the following is not a reason justifying the use of accounting estimates?
a. The valuation or measurement of some accounts is uncertain pending the outcome of future
events.
b. Data about past events cannot be accumulated in a cost-effective manner.
c. Data about future events cannot be accumulated in a cost-effective manner.
d. Data about past events cannot be accumulated in a timely manner.

159. At the conclusion of an audit, an auditor is reviewing the evidence gathered in support of the
financial statements. With regard to the valuation of inventory, the auditor concludes that the
evidence obtained is not sufficient to support management's representations. Which of the following
actions is the auditor most likely to take?
a. Consult with the audit committee and issue a disclaimer of opinion.
b. Consult with the audit committee and issue a qualified opinion.
c. Obtain additional evidence regarding the valuation of inventory.
d. Obtain a statement from management supporting their inventory valuation.

160. When an auditor concludes there is substantial doubt about a continuing audit client's ability to
continue as a going concern for a reasonable period of time, the auditor's responsibility is to:
a. Issue a qualified or adverse opinion, depending upon materiality, due to the possible effects on
the financial statements.
b. Consider the adequacy of disclosure about the client's possible inability to continue as a going
concern.
c. Report to the client's audit committee that management's accounting estimates may need to be
adjusted.
d. Reissue the prior year's auditor's report and add an emphasis of a matter paragraph that
specifically refers to "substantial doubt" and "going concern."

161. An auditor includes a separate paragraph in an otherwise unmodified report to emphasize


that the entity being reported on had significant transactions with related parties. The inclusion of
this separate paragraph:
a. Is considered an "except for" qualification of the opinion.
b. Violates PSAs if this information is already disclosed in footnotes to the financial statements.
c. Necessitates a revision of the opinion paragraph to include the phrase "with the foregoing
explanation."
d. Is appropriate and would not negate the unqualified opinion.

162. A limitation on the scope of an audit sufficient to preclude an unqualified opinion will usually result
when management:
a. Is unable to obtain audited financial statements supporting the entity's investment in a foreign
subsidiary.
b. Refuses to disclose in the notes to the financial statements related party transactions authorized
by the Board of Directors.
c. Does not provide the auditor with an engagement letter specifying the responsibilities of both the
entity and the auditor.
d. Fails to correct a significant deficiency in internal control communicated to those charged with
governance after the prior year's audit.
AT Reviewer Part III Page 22

163. In which of the following situations would an auditor ordinarily choose between expressing an
"except for" qualified opinion or an adverse opinion?
a. The auditor did not observe the entity's physical inventory and is unable to become satisfied as
to its balance by other auditing procedures.
b. The financial statements fail to disclose information that is required by generally accepted
accounting principles.
c. The auditor is asked to report only on the entity's balance sheet and not on the other basic
financial statements.
d. Events disclosed in the financial statements cause the auditor to have substantial doubt about
the entity's ability to continue as a going concern.

164. When an auditor expresses an adverse opinion, the opinion paragraph should include:
a. The principal effects of the departure from generally accepted accounting principles.
b. A direct reference to a separate paragraph disclosing the basis for the opinion.
c. The substantive reasons for the financial statements being misleading.
d. A description of the uncertainty or scope limitation that prevents an unqualified opinion.

165. Under which of the following circumstances would a disclaimer of opinion not be appropriate?
a. The financial statements fail to contain adequate disclosure of related party transactions.
b. The client refuses to permit its attorney to furnish information requested in a letter of audit
inquiry.
c. The auditor is engaged after fiscal year-end and is unable to observe physical inventories or
apply alternative procedures to verify their balances.
d. The auditor is unable to determine the amounts associated with illegal acts committed by the
client's management.

166 The best primary audit evidence regarding year-end bank balances is documented in the
a. Standard bank confirmations c. Bank reconciliations.
b. Interbank transfer schedule d. Bank deposit lead schedule.

167. Which of the following procedures would provide the most reliable audit evidence?
a. Inquiries of the clients internal audit staff held in private
b. Inspection of prenumbered client purchase orders filed in the vouchers payable department
c. Analytical procedures performed by the auditor on the entitys trial balance
d. Inspection of bank statements obtained directly from the clients financial institution

168. Which of the following presumptions is correct about the reliability of audit evidence?
a. Information obtained indirectly from outside sources is the most reliable audit evidence
b. To be reliable, audit evidence should be convincing rather than persuasive
c. Reliability of audit evidence refers to the amount of corroborative evidence obtained
d. An effective internal control structure provides more assurance about the reliability of audit
evidence

169. The most reliable procedure for an auditor to use to test the existence of a clients inventory at an
outside location would be to
a. Observe physical counts of the inventory items
b. Trace the total on the inventory listing to the general ledger inventory account
c. Obtain a confirmation from the client indicating inventory ownership
d. Analytically compare the current-year inventory balance to the prior-year balance

170. An auditor scans a clients investment records for the period just before and just after the year end to
determine that any transfers between categories of investments have been properly recorded. The
primary purpose of this procedure is to obtain evidence about managements financial statement
assertions of
a. Rights and obligations, and existence or occurrence
b. Valuation and allocation, and rights and obligations
c. Existence or occurrence, and classification
d. Classification, and valuation and allocation

171. A client uses a suspense account for unresolved questions whose final accounting has not been
determined. If a balance remains in the suspense account at year-end, the auditor would be most
concerned about
a. Suspense debits that management believes will benefit future operations
b. Suspense debits that the auditor verifies will have realizable value to the client
c. Suspense credits that management believes should be classified as Current liability
d. Suspense credits that the auditor determines to be customer deposits
AT Reviewer Part III Page 23

172. An entitys income statements were misstated due to the recording of journal entries that involved
debits and credits to an unusual combination of expense and revenue accounts. The auditor most
likely could have detected this misstatement by
a. Tracing a sample of journal entries to the general ledger
b. Evaluating the effectiveness of internal control policies and procedures
c. Investigating the reconciliations between controlling accounts and subsidiary records
d. Performing analytical procedures designed to disclose differences from expectations

173. An auditor may achieve audit objective related to particular assertions by


a. Performing analytical procedures c. Preparing auditor working papers.
b. Adhering to a system of quality control d. Increasing the level of detection risk.

174. In addition to evaluating the frequency of deviations in tests of controls, an auditor should also
consider certain qualitative aspects of the deviations. The auditor most likely would give broader
consideration to the implications of a deviation if it was
a. The only deviation discovered in the sample
b. Identical to a deviation discovered during the prior years audit
c. Caused by an employees misunderstanding of instructions
d. Initially concealed by a forged document

178. For which of the following matters should an auditor obtain written management representations?
a. Managements cost-benefit justifications for not correcting internal control weaknesses
b. Managements knowledge of future plans that may affect the price of the entitys stock
c. Managements compliance with contractual agreements that may affect the financial statements
d. Managements acknowledgement of its responsibility for employees violations of laws

179. Which of the following matters most likely would be included in a management representation letter?
a. An assessment of the risk factors concerning the misappropriation of assets
b. An evaluation of the litigation that has been filed against the entity
c. A confirmation that the entity has complied with contractual agreements
d. A statement that all material internal control weaknesses have been corrected

180. To which of the following matters would materiality limits not apply in obtaining written management
representations?
a. The availability of minutes of stockholders and directors meetings
b. Losses from purchase commitments at prices in excess of market value
c. The disclosure of compensating balance arrangements involving related parties
d. Reductions of obsolete inventory to net realizable value

181. To which of the following matters would materiality limits not apply when obtaining written client
representations?
a. Violations of national labor regulations. c. Information about related party transactions.
b. Disclosure of line-of-credit arrangements. d. Instances of fraud involving management.

182. Which of the following statements ordinarily is not included among the written client representations
made by the chief executive officer and the chief financial officer?
a. Sufficient audit evidence has been made available to the auditor to permit the issuance of an
unqualified opinion.
b. There are no unasserted claims or assessments that our lawyer has advised us are probable of
assertion and must be disclosed
c. We have no plans or intentions that may materially affect the carrying value or classification of
assets and liabilities
d. No events have occurred subsequent to the balance sheet date that would require adjustment
to, or disclosure in, the financial statements

183. Which of the following would not be considered an analytical procedure?


a. Estimating payroll expense by multiplying the number of employees by the average hourly wage
rate and the total hours worked.
b. Projecting an error rate by comparing the results of a statistical sample with the actual population
characteristics.
c. Computing accounts receivables turnover by dividing credit sales by the average net
receivables.
d. Developing the expected current year sales based on the sales trend of the prior five years.

184. An auditor compares annual revenues and expenses with similar amount from the prior year and
investigates all changes exceeding 10%. This procedure most likely could indicate that
a. Fourth quarter payroll taxes were properly accrued and recorded, but were not paid until early in
the subsequent year.
b. Unrealized gains from increases in the value of available-for-sale securities were recorded in the
income account for trading securities.
AT Reviewer Part III Page 24

c. The annual provision for uncollectible accounts expenses was inadequate because of worsening
economic conditions.
d. Notice of an increase in property tax rates was received by management, but was not recorded
until early in the subsequent year.

185. An auditor discovered that a clients accounts receivable turnover is substantially lower for the
current year than for the prior year. This may indicate that
a. Obsolete inventory has not yet been reduced to fair market value.
b. There was an improper cutoff of sales at the end of the year.
c. An unusually large receivable was written off near the end of the year.
d. The aging of accounts receivable was improperly performed in both years.

186. An auditor most likely would apply analytical procedures in the overall review stage of an audit to
a. Enhance the auditors understanding of subsequent events.
b. Identify auditing procedures omitted by the staff accountants.
c. Determine whether additional audit evidence may be needed.
d. Evaluate the effectiveness of the internal control activities

187. Which of the following nonfinancial information would an auditor most likely consider in performing
analytical procedures during the planning phase of an audit?
a. Turnover of personnel in the accounting department.
b. Objectivity of audit committee members.
c. Square footage of selling space.
d. Managements plans to repurchase stock.

188. Which of the following factors would least influence an auditors consideration of the reliability of
data for purposes of analytical procedures?
a. Whether the data were processed in a CIS or in a manual accounting system
b. Whether sources within the entity were independent of those who are responsible for the amount
being audited
c. Whether the data were subjected to audit testing in the current or prior year
d. Whether the data were obtained from independent sources outside the entity or from sources
within the entity

189. An auditors analytical procedures performed during the overall review stage indicated that the
clients accounts receivable had doubled since the end of the prior year. However, the allowance for
doubtful accounts as a percentage of accounts receivable remained about the same. Which of the
following client explanations most likely would satisfy the auditor?
a. The client liberalized its credit standards in the current year and sold much more merchandise to
customers with poor credit ratings.
b. Twice as many accounts receivable were written off in the prior year than in the current year.
c. A greater percentage of accounts receivable were currently listed in the more than 90 days
overdue category than in the prior year.
d. The client opened a second retail outlet in the current year and its credit sales approximately
equaled the older, established outlet.

190. In using the work of a expert, an auditor may refer to the expert in the auditors report, if, as a result
of the experts findings, the auditor
a. Because aware of conditions causing substantial doubt about the entitys ability to continue as a
going concern.
b. Desires to disclose the experts findings, which imply that a more thorough audit was performed
c. Is able to corroborate another experts earlier findings that were consistent with managements
representations
d. Discovers significant deficiencies in the design of the entitys internal control that management
does not correct

191. A client decides not to make an auditors proposed adjustments that collectively are not material and
wants the auditor to issue the report based on the unadjusted numbers. Which of the following
statements is correct regarding the financial statement presentation?
a. The financial statements are free from material misstatement, and no disclosure is required in
the notes to the financial statements.
b. The financial statements do not conform with generally accepted accounting principles (GAAP).
c. The financial statements contain unadjusted misstatements that should result in a qualified
opinion.
d. The financial statements are free from material misstatement, but disclosure on the proposed
adjustments is required in the notes to the financial statements.
AT Reviewer Part III Page 25

192. A client is a defendant in a patent infringement lawsuit against a major competitor. Which of the
following items would least likely to be included in the attorneys response to the auditors letter of
inquiry?
a. A description of potential litigation in other matters or related to an unfavorable verdict in the
patent infringement lawsuit.
b. A discussion of case progress and the strategy currently in place by client management to
resolve the lawsuit.
c. An evaluation of the probability of loss and a statement of the amount or range of loss if an
unfavorable outcome is reasonably possible.
d. An evaluation of the ability of the client to continue as a going concern if the verdict is
unfavorable and maximum damages are awarded.

193. Which of the following items would most likely require an adjustment to the financial statements for
the year ended December 31, year 1?
a. Uninsured loss of inventories purchased in year 1 as a result of a flood in year 2
b. Settlement of litigation in year 2 over an event that occurred in year 2
c. Loss on an uncollectible trade receivable recorded in year 1 from a customer that declared
bankruptcy in year 2
d. Proceeds from a capital stock issuance in year 2 which was being approved by the board of
directors in year 1

194. The primary source of information to be reported about litigation, claims and assessments is the
a. Clients lawyer c. Clients management
b. Court records d. Independent auditor.

195. A lawyers response to an auditors inquiry concerning litigation, claims and assessments may be
limited to matters that are considered individually or collectively material to the clients financial
statements. Which parties should reach an understanding on the limits of materiality for this
purpose?
a. The auditor and the clients management. c. The clients management and the lawyer.
b. The clients audit committee and the lawyer. d. The lawyer and the auditor.

196. In connection with an audit of our financial statements, management has prepared, and furnished to
our auditors a description and evaluation of certain contingencies. The foregoing passage most
likely is from a(an)
a. Audit inquiry letter to legal counsel c. Audit committees communication to auditor.
b. Management representation letter d. Financial statement footnote disclosure

197. In creating lead schedules for an audit engagement, a CPA often uses automated work paper
software. What client information is needed to begin this process?
a. Interim financial information such as third quarter sales, net income, and inventory and
receivables balances
b. Specialized journal information such as the invoice and purchase order numbers of the last few
sales and purchases of the year
c. General ledger information such as account numbers, prior-year account balances, and current-
year unadjusted information
d. Adjusting entry information such as deferrals and accruals, and reclassification journal entries

198. There have been no communications from regulatory agencies concerning non-compliance with, or
deficiencies in, financial reporting practices that could have a material effect on the financial
statements. The foregoing passage is most likely from a
a. Report on internal control c. Management representation letter.
b. Special report d. Letter of underwriters.

199. Franz, CPA is preparing an audit program for the purpose of ascertaining the occurrence of
subsequent events that may require adjustment or disclosure essential to a fair presentation of the
financial statements in conformity with financial reporting framework. Which of the following
procedures would least appropriate for this purpose?
a. Confirm as of the completion of fieldwork accounts receivable which have increased significantly
from the year-end date.
b. Read the minutes of the board of directors.
c. Inquire of management concerning events, which may have occurred.
d. Obtain a lawyers letter as of the completion of fieldwork.

199. In performing a count of negotiable securities, an auditor records the details of the count on a
security count worksheet. What other information is usually included on this worksheet?
a. An acknowledgment by a client representative that the securities were returned intact.
b. An analysis of realized gains and losses from the sale of securities during the year.
c. An evaluation of the clients internal control concerning physical access to the securities.
d. A description of the clients procedures that prevent the negotiation of securities by just one
person.
AT Reviewer Part III Page 26

200. Which of the following procedures most likely could assist an auditor in identifying related party
transactions?
a. Performing tests of controls concerning the segregation of duties.
b. Evaluating the reasonableness of managements accounting estimates.
c. Reviewing confirmations on compensating balance arrangements.
d. Scanning the accounting records for recurring transactions.

201. When auditing related party transactions, an auditor places primary emphasis on
a. Ascertaining the rights and obligations of the related parties
b. Confirming the existence of the related parties
c. Verifying the valuation of the related party transactions
d. Evaluating the disclosure of the related party transactions

202. An auditor searching for related party transactions should obtain an understanding of each
subsidiarys relationship to the total entity because
a. The business structure may be deliberately designed to obscure related party transactions
b. Intercompany transactions may have been consummated on terms equivalent to arms-length
transactions
c. This may reveal whether particular transactions would have taken place if the parties had not
been related
d. This may permit the audit of intercompany account balances to be performed as of concurrent
dates

203. After determining that a related party transaction has, in fact, occurred, an auditor should
a. Add a separate paragraph to the auditors standard report to explain the transaction
b. Perform analytical procedures to verify whether similar transactions occurred, but were not
recorded
c. Obtain an understanding of the business purpose of the transaction
d. Substantiate that the transaction was consummated on terms equivalent to an arms length
transaction

204. An auditor most likely would modify an unqualified opinion if the entitys financial statements include
a footnote on related party transactions
a. Disclosing loans to related parties at interest rates significantly below prevailing market rates
b. Describing an exchange of real estate for similar property in a nonmonetary related party
transaction
c. Stating that a particular related party transaction occurred on terms equivalent to those that
would have prevailed in an arm-length transaction
d. Presenting the peso-volume of related party transactions and the effects of any change in the
method of establishing terms from prior periods

205. A CPA firm is completing the fieldwork for an audit of Plenty Co. for the current year ended
December 31. The manager in charge of the audit is performing the final steps in the evidence
accumulation phase of the audit and notes that there have been several changes in Plenty Co.
during the year under audit. Which of the following items would indicate there could be substantial
doubt about Plentys ability to continue as a going concern for a reasonable period of time?
a. Cash infusion by a venture capital firm
b. Recurring working capital shortages
c. A lack of significant contracts with new customers
d. Term debit refinanced with a new bank

Substantive Testing of Sales and Receivables

206. If the objective of a test of details is to detect overstatements of sales, the auditor should compare
transactions in the
a. Cash receipts journal with the sales journal.
b. Sales journal with the cash receipts journal
c. Source documents with the accounting records
d. Accounting records with the source documents

207. PSA 505, The External Confirmation, defines confirmation as the process of obtaining and
evaluating a direct communication from a third party in response to a request for information About a
particular item affecting financial statement assertions. Two assertions for which confirmation of
accounts receivable balances provides primary evidence are
a. Completeness and valuation
b. Valuation and rights and obligations.
c. Rights and obligations and existence
d. Existence and completeness.
AT Reviewer Part III Page 27

207. Under which of the following circumstances would an auditor be most likely to intensify an audit of
P20,000 petty cash fund?
a. Petty cash vouchers are not prenumbered.
b. The custodian endorses reimbursement checks.
c. Reimbursement occurs twice each week.
d. The custodian occasionally uses the petty cash funds to cash employee checks.

208. An auditor confirms a representative number of open accounts receivable as of December 31 and
investigates respondents exceptions and comments. By this procedure, the auditor would be most
likely to learn of which of the following?
a. One of the cashiers has been covering a personal embezzlement by lapping.
b. One of the sales clerks has not been preparing charge slips for credit sales to family and friends.
c. One of the computer control clerks has been removing all sales invoices applicable his account
from the data file.
d. The credit manager has misappropriated remittances from customers whose accounts have been
written off.

209. An auditor suspects that a clients cashier is misappropriating cash receipts for personal use by
lapping customer checks received in the mall. In attempting to uncover this embezzlement scheme,
the auditor most likely would compare the
a. Dates checks are deposited per bank statements with the dates remittance credits are recorded.
b. Daily cash summaries with the sums of the cash receipts journal entries.
c. Individual bank deposit slips with the details of the monthly bank statements.
d. Dates uncollectible accounts are authorized to be written off with the dates the write-offs are
actually recorded.

210. For the fiscal year ending December 31, previous year and the current year, Abundant Co. has net
sales of P1,000,000 and P2,000,000; average gross receivables of P100,000 and P300,000; and
allowance for uncollectible accounts receivable of P30,000 and P50,000, respectively. If the
accounts receivable turnover and the ratio of allowance for uncollectible accounts receivable to
gross accounts receivable are calculated, which of the following best represents the conclusions to
be drawn?
a. Accounts receivable turnovers are 10.0 and 6.6 and the ratios of uncollectible accounts
receivable to gross accounts receivable are 0.30 and 0.16, respectively. Examine allowance for
possible overstatement of the allowance
b. Accounts receivable turnovers are 10.0 and 6.6 and the ratios of uncollectible accounts
receivable to gross accounts receivable are 0.30 and 0.16 respectively. Examine allowance for
possible understatement of the allowance
c. Accounts receivable turnovers are 14.3 and 8.0 and the ratios of uncollectible accounts
receivable to gross accounts receivable is 0.42and 0.20 respectively. Examine allowance for
possible overstatement of the allowance
d. Accounts receivable turnovers are 14.3 and 8.0 and the ratios of uncollectible accounts
receivable to gross accounts receivable is 0.42and 0.20 respectively. Examine allowance for
possible understatement of the allowance

Substantive Testing of Cash

212. The best evidence regarding year-end bank balances is documented in the
a. Cutoff bank statement. c. Interbank transfers schedule
b. Bank reconciliations. d. Bank deposit lead schedule.

213. Which of the following sets of information does an auditor usually confirm on one form?
a. Accounts payable and purchase commitments
b. Cash in bank and collateral for loans.
c. Inventory on consignment and contingent liabilities.
d. Accounts receivable and accrued interest receivable

214. An independent auditor asked a clients internal auditor to assist in preparing a standard financial
institution confirmation request for a payroll account that had been closed during the year under
audit. After the internal auditor prepared the form, the controller signed it and mailed it to the bank.
What was the major flaw in this procedure?
a. The internal auditor did not sign the form c. The form was prepared by internal auditor
b. The form was mailed by the controller d. The account was closed so the balance was zero.

215. An auditor should test bank transfers for the last part of the audit period and the first part of the
subsequent period to detect whether
a. The cash receipts journal was held open for a few days after year-end
b. The last checks recorded before year-end were actually mailed by year-end
c. Cash balances were overstated because of kiting
d. Any unusual payments for receipts from related parties occurred
AT Reviewer Part III Page 28

Substantive Testing of Accounts Payable and Purchases

216. Which of the following is a substantive test that an auditor most likely would perform to verify the
existence and valuation of recorded accounts payable?
a. Investigating the open purchase order file to ascertain that prenumbered purchase orders are
used and accounted for.
b. Receiving the clients mail, unopened, for a reasonable period of time after year-end to search for
unrecorded vendors invoices.
c. Vouching selected entries in the accounts payable subsidiary ledger to purchase orders and
receiving reports.
d. Confirming accounts payable balances with known suppliers who have zero balances.

217. To determine whether accounts payable are complete, an auditor performs a test to verify that all
merchandise received is recorded. The population of documents for this test consists of all
a. Payment vouchers. c. Purchase requisitions,
b. Receiving reports. d. Vendors invoices.

218. An auditor traced a sample of purchase orders and the related receiving reports to the purchases
journal and the cash disbursements journal. The purpose of this substantive audit procedure most
likely was to
a. Identify unusually large purchases that should be investigated further.
b. Verify the cash disbursements were for goods actually received.
c. Determine that purchases were properly recorded.
d. Test whether payments were for goods actually ordered.

219. An auditor performs a test to determine whether all merchandise for which the client was billed was
received. The population for this test consists of all
a. Merchandise received c. Canceled checks
b. Vendors invoices d. Receiving reports

220. In auditing accounts payable, an auditors procedure most likely would focus primarily on
managements assertion of
a. Existence or occurrence c. Completeness
b. Presentation and disclosure d. Valuation or allocation

221. Which of the following procedures would an auditor most likely perform in searching for unrecorded
liabilities?
a. Trace a sample of accounts payable entries recorded just before year-end to the unmatched
receiving report file
b. Compare a sample of purchase orders issued just after year-end with the year-end accounts
payable trial balance
c. Vouch a sample of cash disbursements recorded just after year-end to receiving reports and
vendor invoices
d. Scan the cash disbursement entries recorded just before year-end for indications of unusual
transactions

222. An auditors purpose in reviewing the renewal of a notes payable shortly after the balance sheet date
most likely is to obtain evidence concerning managements assertions about
a. Existence or occurrence c. Completeness
b. Presentation and disclosure d. Valuation or allocation

223. Which of the following procedures would an auditor least likely perform before the balance sheet
date?
a. Confirmation of accounts payable c. Assessment of control risk.
b. Observation of merchandise inventory d. Identification of related parties.

224. When using confirmations to provide evidence about the completeness assertion for accounts
payable, the appropriate population most likely is
a. Vendors with whom the entity has previously done business
b. Amounts recorded in the accounts payable subsidiary ledger
c. Payees of checks drawn in the month after the year-end
d. Invoices filed in the entitys open invoice file

225. An auditor suspects that certain client employees are ordering merchandise for themselves over the
Internet without recording the purchase or receipt of the merchandise. When vendors invoices
arrive, one of the employees approves the invoices for payment. After the invoices are paid, the
employee destroys the invoices and the related vouchers. In gathering evidence regarding the
fraud, the auditor most likely would select items for testing from the file of all
a. Cash disbursements c. Approved vouchers.
b. Approved vouchers d. Vendors invoices
AT Reviewer Part III Page 29

226. Cutoff tests designed to detect purchases made before the end of the year that have been recorded
in the subsequent year most likely would provide assurance about managements assertion of
a. Existence or occurrence c. Completeness
b. Presentation and disclosure d. Valuation or allocation

227. An internal control narrative indicates that an approved voucher is required to support every check
request for payment of merchandise. Which of the following procedures provides the greatest
assurance that this control is operating effectively?
a. Select and examine vouchers and ascertain that the related canceled checks are dated no later
than the vouchers
b. Select and examine vouchers and ascertain that the related canceled checks are dated no
earlier than the vouchers
c. Select and examine canceled checks and ascertain that the related vouchers are dated no
earlier than the checks
d. Select and examine canceled checks and ascertain that the related vouchers are dated no later
than the checks

228. When performing procedures to test assertions about purchases, an auditor vouches a sample of
entries in the voucher register to the supporting documents. Which assertion would this procedure
most likely support?
a. Completeness c. Valuation or allocation.
b. Existence or occurrence d. Rights and obligations

Substantive Testing of Inventory

229. The element of the audit-planning process most likely to be agreed upon with the client before
implementation of the audit strategy is the determination of the
a. Evidence to be gathered to provide a sufficient basis for the auditors opinion
b. Procedures to be undertaken to discover litigation, claims, and assessments
c. Pending legal matters to be included in the inquiry of the clients attorney
d. Timing of inventory observation procedures to be performed

230. When auditing inventories, an auditor would least likely verify that
a. All inventory owned by the client is on hand at the time of the count
b. The client has used proper inventory pricing
c. The financial statement presentation of inventories is appropriate
d. Damaged goods and obsolete items have been properly accounted for

231. A client maintains perpetual inventory records in quantities and in peso. If the assessed level of
control risk is high, an auditor would probably
a. Apply gross profit tests to ascertain the reasonableness of the physical counts.
b. Increase the extent of tests of controls relevant to the inventory cycle.
c. Request the client to schedule the physical inventory count at the end of the year.
d. Insist that the client perform physical counts of inventory items several times during the year.

232. To gain assurance that all inventory items in a clients inventory listing schedule are valid, an auditor
most likely would vouch
a. Inventory tags noted during the auditors observation to items listed in the inventory listing
schedule.
b. Inventory tags noted during the auditors observation to items listed in receiving reports and
vendors invoices.
c. Items listed in the inventory listing schedule to inventory tags and the auditors recorded count
sheets.
d. Items listed in receiving reports and vendors invoices to the inventory listing schedule.

233. An auditor selected items for test counts while observing a clients physical inventory. The auditor
then traced the test counts to the clients inventory listing. This procedure most likely obtained
evidence concerning managements assertion of
a. Rights and obligations c. Existence or occurrence
b. Completeness d. Valuation

234. Which of the following audit procedures probably would provide the most reliable evidence
concerning the entitys assertion of rights and obligations related to inventories?
a. Trace test counts noted during the entitys physical count to the entitys summarization of
quantities.
b. Inspect agreements to determine whether any inventory is pledged as collateral or subject to any
liens.
c. Select the last few shipping advices used before the physical count and determine whether the
shipments were recorded as sales.
d. Inspect the open purchase order file for significant commitments that should be considered for
disclosure.
AT Reviewer Part III Page 30

235. To measure how effectively an entity employs its resources, an auditor calculates inventory turnover
by dividing average inventory into
a. Net sales c. Operating income
b. Cost of goods sold d. Gross sales

236. An auditor most likely would analyze inventory turnover rates to obtain evidence concerning
managements assertions about
a. Existence or occurrence c. Presentation and disclosure.
b. Rights and obligations d. Valuation or allocation

237. An auditor most likely would make inquiries of production and sales personnel concerning possible
obsolete or slow-moving inventory to support managements financial statement assertion of
a. Valuation or allocation c. Existence or occurrence
b. Rights and obligations d. Presentation and disclosure

238. An auditor most likely would inspect loan agreements under which an entitys inventories are
pledged to support managements financial statement assertion of
a. Existence or occurrence c. Presentation and disclosure
b. Completeness d. Valuation or allocation

239. An auditor concluded that no excessive costs for an idle plant were charged to inventory. This
conclusion most likely related to the auditors objective to obtain evidence about the financial
statement assertions regarding inventory, including presentation and disclosure and
a. Valuation and allocation c. Existence or occurrence
b. Completeness d. Rights and obligations.

240. Which of the following auditing procedures most likely would provide assurance about a
manufacturing entitys inventory valuation?
a. Testing the entitys computation of standard overhead rates
b. Obtaining confirmation of inventories pledged under loan agreements
c. Reviewing shipping and receiving cutoff procedures for inventories
d. Tracing test counts to the entitys inventory listing

Substantive Testing of Property, Plant and Equipment

241. Which of the following combinations of procedures would an auditor most likely perform to obtain
evidence about fixed asset additions?
a. Inspecting documents and physically examining assets.
b. Recomputing calculations and obtaining written management representations.
c. Observing operating activities and comparing balances with prior period balances.
d. Confirming ownership and corroborating transactions through inquiries of client personnel.

242. In testing plant and equipment balances, an auditor may inspect new additions listed on the analysis
of plant and equipment. This procedure is designed to obtain evidence concerning managements
assertions about
Existence or Occurrence Presentation and Disclosure
a. Yes Yes
b. Yes No
c. No Yes
d. No No

243. A weakness in internal control over recording retirements of equipment may cause an auditor to
a. Inspect certain items of equipment in the plant and trace those items to the accounting records.
b. Review the subsidiary ledger to ascertain whether depreciation was taken on each item of
equipment during the year.
c. Trace additions to the other assets account to search for equipment that is still on hand but no
longer being used.
d. Select certain items of equipment from the accounting records and locate them in the plant.

244. Determining that proper amounts of depreciation are expensed provides assurance about
managements assertions of valuation or allocation and
a. Presentation and disclosure
b. Completeness
c. Rights and obligations
d. Existence or occurrence

245. In performing a search for unrecorded retirements of fixed assets, an auditor most likely would
a. Inspect the property ledger and the insurance and tax records, and then tour the clients
facilities.
b. Tour the clients facilities, and then inspect the property ledger and the insurance and tax
records.
c. Analyze the repair and maintenance account, and then tour the clients facilities.
d. Tour the clients facilities, and then analyze the repair and maintenance account.
AT Reviewer Part III Page 31

246. Which of the following explanations most likely would satisfy an auditor who questions management
about significant debits to the accumulated depreciation accounts?
a. The estimated remaining useful lives of plant assets were revised upward.
b. Plant assets were retired during the year.
c. The prior years depreciation expense was erroneously understated
d. Overhead allocations were revised at year-end.

247. When auditing prepaid insurance, an auditor discovers that the original insurance policy, on plant
equipment is not available for inspection. The policys absence most likely indicates the possibility of
a(n)
a. Insurance premium due but not recorded.
b. Deficiency in the coinsurance provision.
c. Lien on the plant equipment.
d. Understatement of insurance expense.

248. An auditor analyzes repairs and maintenance accounts primarily to obtain evidence in support of the
audit assertion that all
a. Non-capitalizable expenditures for repairs and maintenance have been recorded in the proper
period.
b. Expenditures for property and equipment have been recorded in the proper period.
c. Non-capitalizable expenditures for repairs and maintenance have been properly charged to
expense.
d. Expenditure for property and equipment have not been charged to expense

249. In auditing intangible assets, an auditor most likely would review or recomputed amortization and
determine whether the amortization period is reasonable in support of managements financial
statement assertion of
a. Valuation or allocation.
b. Existence or occurrence.
c. Completeness.
d. Rights and obligations.

Substantive Testing of Investments

250. Which of the following pairs of accounts would an auditor most likely analyze on the same working
paper?
a. Notes receivable and interest income
b. Accrued interest receivable and accrued interest payable
c. Notes payable and notes receivable
d. Interest income and interest expense

251. An auditor would most likely verify the interest earned on bond investments by
a. Vouching the receipt and deposit of interest checks
b. Confirming the bond interest rate with the issuer of the bonds
c. Recomputing the interest earned on the basis of face amount, interest rate, and period held
d. Testing internal controls relevant to cash receipts

252. An auditor usually tests the reasonableness of dividend income from investments in publicly held
companies by computing the amounts that should have been received by referring to
a. Dividend record books produced by investment advisory services.
b. Stock indentures published by corporate transfer agents.
c. Stock ledgers maintained by independent registrars.
d. Annual audited financial statements issued by the investee companies.

253. A client has a large and active investment portfolio that is kept in a bank safe-deposit box. If the
auditor is unable to count the securities at the balance sheet date, the auditor most likely will
a. Request the bank to confirm to the auditor the contents of the safe-deposit box at the balance
sheet date.
b. Examine supporting evidence for transactions occurring during the year.
c. Count the securities at a subsequent date and confirm with the bank whether securities were
added or removed since the balance sheet date.
d. Request the client to have the bank seal the safe-deposit box until the auditor can count the
securities at a subsequent date.
AT Reviewer Part III Page 32

254. In performing a count of negotiable securities, an auditor records the details of the count on a
security count worksheet. What other information is usually included on this worksheet?
a. An acknowledgment by a client representative that the securities were returned intact
b. An analysis of realized gains and losses from the sale of securities during the year
c. An evaluation of the clients internal control concerning physical access to the securities
d. A description of the clients procedures that prevent the negotiation of securities by just one
person

255. An auditor testing long-term investments would ordinarily use analytical procedures to ascertain the
reasonableness of the
a. Existence of unrealized gains or losses
b. Completeness of recorded investment income
c. Classification as available-for-sale or trading securities
d. Valuation of trading securities

256. In confirming with an outside agent, such as a financial institution, that the agent is holding
investment securities in the clients name, an auditor most likely gathers evidence in support of
managements financial statement assertions of existence or occurrence and
a. Valuation or allocation
b. Rights and obligations
c. Completeness
d. Presentation and disclosure

257. An auditor scans a clients investment records, for the period just before and just after the year-end to
determine that any transfers between categories of investments have been properly recorded. The
primary purpose of this procedure is to obtain evidence about managements financial statement
assertions of
a. Rights and obligations, and existence or occurrence
b. Valuation or allocation, and rights and obligations
c. Existence or occurrence, and presentation and disclosure
d. Presentation and disclosure, and valuation or allocation

258. To satisfy the valuation assertion when auditing an investment accounted for by the equity method,
an auditor most likely would
a. Inspect the stock certificates evidencing the investment
b. Examine the audited financial statements of the investee company
c. Review the brokers advice or canceled check for the investments acquisition
d. Obtain market quotations from financial newspapers or periodicals

259. Auditors may need to plan and perform auditing procedures for financial statement assertions about
derivatives and hedging activities. Which of the following substantive procedures most clearly tests
the completeness assertion about derivatives?
a. Assessing the reasonableness of the use of an option-pricing model
b. Determining whether changes in the fair value of derivatives designated and qualifying as
hedging instruments have been reported in earnings or in other comprehensive income
c. Requesting counterparties to provide information About them, such as whether side agreements
have been made
d. Physically inspecting the derivative contract

Substantive Testing of Long-Term Debt

260. An auditors program to audit long-term debt should include steps that require
a. Examining bond trust indentures.
b. Inspecting the accounts payable subsidiary ledger.
c. Investigating credits to the bond interest income account.
d. Verifying the existence of the bondholders.

261. In auditing for unrecorded long-term bonds payable, an auditor most likely will
a. Perform analytical procedures on the bond premium and discount accounts.
b. Examine documentation of assets purchased with bond proceeds for liens.
c. Compare interest expense with the bond payable amount for reasonableness.
d. Confirm the existence of individual bondholders at year-end.

Substantive Testing of Equity


AT Reviewer Part III Page 33

262. During an audit of a companys equity accounts, the auditor determines whether restrictions have
been imposed on retained earnings resulting from loans, agreements, or state law. This audit
procedure most likely is intended to verify managements assertion of
a. Existence or occurrence
b. Completeness
c. Valuation or allocation
d. Presentation and disclosure

263. When a clients company does not maintain its own stock records, the auditor should obtain written
confirmation from the transfer agent and registrar concerning
a. Restrictions on the payment of dividends.
b. The number of shares issued and outstanding.
c. Guarantees of preferred stock liquidation value.
d. The number of shares subject to agreements to repurchase.

264. An auditor usually obtains evidence of a companys equity transactions by reviewing its
a. Minutes of board of directors meetings.
b. Transfer agents records.
c. Canceled stock certificates.
d. Treasury stock certificate book.

Substantive Testing of Payroll

265. An auditor vouched data for a sample of employees in a payroll register to approve clock card data to
provide assurance that
a. Payments to employees are computed at authorized rates
b. Employees work the number of hours for which they are paid
c. Segregation of duties exist between the preparation and distribution of the payroll
d. Internal controls relating to unclaimed payroll checks are operating effectively

266. When control risk is assessed as low for assertions related to payroll, substantive tests of payroll
balances most likely would be limited to applying analytical procedures and
a. Observing the distribution of paychecks
b. Footing and cross footing the payroll register
c. Inspecting payroll tax returns
d. Recalculating payroll accruals

267. An auditor most likely would perform substantive tests of details on payroll transactions and balances
when
a. Cutoff tests indicate a substantial amount of accrued payroll expense
b. The assessed level of control risk relative to payroll transactions is low
c. Analytical procedures indicate unusual fluctuations in recurring payroll entries
d. Accrued payroll expense consists primarily of unpaid commissions

268. In auditing payroll when control risk is assessed as low, an auditor most likely will
a. Verify that checks representing unclaimed wages are mailed.
b. Trace individual employee deductions to entity journal entries.
c. Observe entity employees during a payroll distribution.
d. Compare payroll costs with entity standards or budgets.

269. An auditor most likely increases substantive tests of payroll when


a. Payroll is extensively audited by the national government.
b. Payroll expense is substantially higher than in the prior year.
c. Overpayments are discovered in performing tests of details.
d. Employees complain to management About too much overtime.

270. Which of the following circumstances most likely will cause an auditor to suspect an employee payroll
fraud scheme?
a. There are significant unexplained variances between standard and actual labor cost.
b. Payroll checks are disbursed by the same employee each payday.
c. Employee time cards are approved by individual departmental supervisors.
d. A separate payroll bank account is maintained on an imprest basis

271, When an auditor concludes there is substantial doubt about a continuing audit clients ability to
continue as a going concern for a reasonable period of time, the auditors responsibility is to
a. Express a qualified or adverse opinion, depending upon materiality, due to the possible effects
on the financial statements.
b. Consider the adequacy of disclosure about the clients possible inability to continue as a going
concern.
c. Report to the clients audit committee that managements accounting estimates may need to be
adjusted.
d. Reissue the prior years auditors report and an explanatory paragraph that specifically refers to
substantial doubt and going concern.
AT Reviewer Part III Page 34

Evidence Sampling

272. If all other factors in a sampling plan are held constant, changing the measure of tolerable error to a
smaller value would cause the sample size to be:
a. Smaller. b. Larger.
c. Unchanged. d. Indeterminate.

273. Precision is defined as the range (plus or minus) within which the true answer most likely falls.
Reliability, also referred to as the confidence level, is the likelihood that the sample range contains
the true value. Of the following statements concerning precision and reliability, which one is not
true?
a. Precision is set by the auditor and is a function of materiality and risk.
b. The narrower the range of precision, the smaller the sample size.
c. An increase in control risk, other parameters remaining unchanged, causes a narrowing of the
precision range.
d. Overall audit risk is the complement of reliability.

274. An auditor established a P60,000 tolerable misstatement for an asset with an account balance of
P1,000,000. The auditor selected a sample of every twentieth item from the population that
represented the asset account balance and discovered overstatements of P3,700 and
understatement of P200. Under these circumstance the auditor most likely would conclude that
a. There is an unacceptably high risk that the actual misstatement in the population exceed the
tolerable misstatement because the total projected misstatement is more than the tolerable
misstatement,
b. There is an unacceptably high risk that the tolerable misstatement exceeds the sum of actual
overstatements and understatements.
c. The asset account is fairly stated because the total projected misstatement is less than the
tolerable misstatement.
d. The asset account is fairly stated because the tolerable misstatement exceeds the net of
projected actual overstatements and understatements.

275. An auditor is performing substantive tests of pricing and extensions of perpetual inventory balances
consisting of a large numbers of items. Past experience indicates numerous pricing and extension
errors. Which of the following statistical sampling approaches is most appropriate?
a. Unstratified mean-per-unit.
b. Probability-proportional-to-size.
c. Stop or go,
d. Ratio or difference estimation.

276. In addition to evaluating the frequency of deviations in tests of controls, an auditor should also
consider certain qualitative aspects of the deviations. The auditor most likely would give broader
consideration to the implications of a deviation if it was
a. The only deviation discovered in the sample.
b. Identical to a deviation discovered during the prior years audit.
c. Caused by an employees misunderstanding of instructions.
d. Initially concealed by a forged document.

277, Stratified mean-per-unit (MPU) sampling is a statistical technique that may be more efficient than
unstratified MPU because it usually
a. May be applied to populations in which many monetary errors are expected to occur.
b. Produces an estimate having a desired level of precision with a smaller sample size.
c. Increases the variability among items in a stratum by grouping sampling units with similar
characteristics.
d. Yields a weighted sum of the strata standard deviations that is greater than the standard
deviation of the population.

278, Which of the following statements is true concerning probability-proportional-to-size (PPS) sampling
also known as monetary-unit sampling?
a. The sampling distribution should approximate the normal distribution.
b. Overstated units have a lower probability of sample selection that units that are understated.
c. The auditor controls the risk of incorrect acceptance by specifying that risk level for the
sampling plan.
d. The sampling interval is calculated by dividing the number of physical units in the population by
the sample size.

279. In comparison with classical variables sampling, which of the following is an advantage of
probability-proportional-to-size sampling (PPS)?
a. PPS sampling automatically results in a stratified sample.
b. PPS sampling results in a smaller sample size if many differences are expected between audited
and recorded amounts.
c. PPS sampling is particularly appropriate when understatement errors are expected.
AT Reviewer Part III Page 35

d. PPS sampling is less likely to overstate the allowance for sampling risk when errors are found in
the sample,

280. Which of the following most likely would be an advantage in using classical variables sampling rather
than probability-proportional-to-size sampling?
a. An estimate of the standard deviation of the populations recorded amounts is not required.
b. The auditor rarely needs the assistance of a computer program to design an efficient sample.
c. Inclusion of zero and negative balances usually does not require special design considerations.
d. Any amount that is individually significant is automatically identified and selected.

281. Probability-proportional-to-size sampling would likely result in selecting a sample with characteristics
roughly equivalent to:
a. A classical variables sampling plan stratified by peso amount.
b. Difference estimation.
c. Ratio estimation.
d. Nonstatistical sampling.

282. PPS sampling is less efficient if


a. Computerized account balances are being audited.
b. Statistical inferences are to be made.
c. The audit objective is oriented to understatements.
d. The amount contains a large number of transactions.

PSA 700 (Redrafted) Forming an Opinion and Reporting on Financial Statements

283. A CPA concludes that the unaudited financial statements on which the CPA is disclaiming an opinion
are not in conformity with generally accepted accounting principles (GAAP) because management
has failed to capitalize leases. The CPA suggests appropriate revisions to the financial statements
but management refuses to accept the CPAs suggestions. Under these circumstances, the CPA
ordinarily would
a. Express limited assurance that no other material modifications should be made to the financial
statements
b. Restrict the distribution of the CPAs report to management and the entitys board of directors
c. Issue a qualified opinion or adverse opinion depending on the materiality of the departure from
GAAP
d. Describe the nature of the departure from GAAP in the CPAs report and state the effects on the
financial statements, if practicable

283. A financial statement audit client has an information processing system where all information system
is transmitted, processed and maintained electronically. Because the company has very little
tangible data available, the auditor has decided that it will be impossible to do enough substantive
testing to reduce the risk of material misstatement to an acceptably low level. What should the
auditor do in this situation?
a. Express a qualified opinion or disclaim an opinion.
b. Withdraw from the engagement.
c. Perform additional tests of controls to reduce the risk of material misstatement.
d. Recommend that the client convert a portion of the information processing system to a manual
system so backup documentation can be made available to the auditor.

84. In which of the following situations would an auditor ordinarily choose between expressing a qualified
opinion or an adverse opinion?
a. The auditor did not observe the entity's physical inventory and is unable to become satisfied about
its balance by other auditing procedures.
b. Conditions that cause the auditor to have substantial doubt about the entity's ability to continue as a
going concern are inadequately disclosed.
c. There has been a change in accounting principles that has a material effect on the comparability of
the entity's financial statements.
d. The auditor is unable to apply necessary procedures concerning an investor's share of an investee's
earnings recognized on the equity method.

285. The main purpose of the auditor/client conference held at the close of audit field work is to
a. Discuss unresolved matters and audit fee arrangements.
b. Review proposed audit adjustments, internal control weaknesses, and needed disclosures, and
agree on the type of audit report to be rendered.
c. Discuss areas of major audit risk and use of client personnel to assist in high risk areas.
d. Arrange for a meeting with outside legal counsel for the purpose of discussing pending litigation.

286. On March 1, Stevens, CPA, expressed an unqualified opinion on the financial statements of
Provident Co. On July 1, Stevens internal inspection program discovered that engagement
personnel failed to observe Providents physical inventory. Stevens believes that his omission
AT Reviewer Part III Page 36

impairs Stevens ability to support the unqualified opinion. If Providents creditors are currently
relying on Stevens opinion, Stevens should first
a. Request Providents management to communicate to its creditors that Stevens opinion should
not be relied on
b. Reissue Stevens auditors report with an explanatory paragraph describing the departure from
PSA.
c. Undertake to apply the alternative procedures that would provide a satisfactory basis for
Stevens opinion
d. Advise Providents board of directors to disclose this development in its next interim report

287. After issuing a report, an auditor concludes that an auditing procedure considered necessary at the
time of the audit was omitted from the audit. The auditor should first
a. Undertake to apply the omitted procedure or alternative procedures that would provide a
satisfactory basis for the auditors opinion
b. Assess the importance of the omitted procedure to the auditors ability to support the opinion
expressed on the financial statements taken as a whole
c. Notify the audit committee or the board of directors that the audit opinion can no longer be relied
upon
d. Review the results of other procedures that were applied to compensate for the one omitted or to
make its omission less important

288. The auditor concludes that there is a material inconsistency in the other information in an annual
report to shareholders containing audited financial statements. If the client refuses to revise or
eliminate the material inconsistency, the auditor should
a. Revise the auditors report to include a separate explanatory paragraph describing the material
inconsistency
b. Consult with a party whose advice might influence the client, such as the clients legal counsel
c. Issue a qualified opinion after discussing the matter with the clients board of directors
d. Consider the matter closed since the other information is not in the audited financial statements

289 An auditor would be most likely to consider expressing a qualified opinion if the clients financial
statements include a footnote on related party transactions that
a. Lists the amounts due from related parties including the terms and manner of settlement
b. Discloses compensating balance arrangements maintained for the benefit of related parties
c. Represents that certain transactions with related parties were consummated on terms equally as
favorable as would have been obtained in transactions with unrelated parties
d. Presents the dollar volume of related party transactions and the effects of any change in the
method of establishing terms from that of the prior period

290. An annual shareholders report includes audited financial statements and contains a management
report asserting that the financial statements are the responsibility of management. Is it permissible
for the auditors report to refer to the management report?
a. No, because the reference may lead to the belief that the auditor is providing assurances about
managements representations.
b. No, because the auditor has no responsibility to read the other information in a document
containing audited financial statements.
c. Yes, provided the reference is included in a separate explanatory paragraph of the auditors
report.
d. Yes, provided the auditor reads the management report and discovers no material
misrepresentation of fact.

291. Ronald, CPA, suspects that a pervasive scheme of illegal bribes exists throughout the operations of
Nationwide Import-Export, Inc., a new audit client. Ronald notified the audit committee and
Nationwide's legal counsel, but neither could assist Ronald in determining whether the amounts
involved were material to the financial statements or whether senior management was involved in the
scheme. Under these circumstances, Ronald should:
a. Express an unqualified opinion with an emphasis of matter paragraph.
b. Disclaim an opinion on the financial statements.
c. Express an adverse opinion on the financial statements.
d. Issue a special report regarding the illegal bribes.

292. Justine, CPA, completed the field work of the audit of Apollos December 31, year 1 financial
statements on March 6, year 2. A subsequent event requiring adjustment of the year 1 financial
statements occurred on April 10, year 2, and came to Justines attention on April 24, year 2. The
subsequent event occurred prior to the issuance of the related financial statements. If the adjustment
is made without disclosure of the event, Justines report ordinarily should be dated
a. March 6, year 2
b. April 10, year 2
c. April 24, year 2
d. Using dual dating
AT Reviewer Part III Page 37

293. On Aug 13, a CPA completed field work on an engagement to audit financial statements for the year
ended June 30. On August 27, an event came to the CPAs attention that should be disclosed in the
notes to the financial statements. The event was properly disclosed by the entity, but the CPA
decided not to dual-date the auditors report and dated the report August 27. Under these
circumstances, the CPA was taking responsibility for
a. All subsequent events that occurred through August 27
b. Only the specific subsequent event disclosed by the entity
c. All subsequent events that occurred through August 13 and the specific subsequent event
disclosed by the entity
d. Only the subsequent events that occurred through August 13

294. 1st The auditors opinion is an assurance as to the future validity of the entity or an opinion to the
efficiency and effectiveness with which management conducts the affairs of the entity
2nd The auditors opinion helps establish the credibility of financial statements
a. First statement is false, second statement is true
b. First statement is true, second statement is false
c. Both statements are true
d. Both statements are false

295. A limitation on the scope of an audit sufficient to preclude an unqualified opinion will usually result when
management
a. Presents financial statements that are prepared in accordance with the cash receipts and
disbursements basis of accounting.
b. States that the financial statements are not intended to be presented in conformity with generally
accepted accounting principles.
c. Does not make the minutes of the board of directors meetings to the auditor.
d. Asks the auditor to report on the balance sheet and not on the other basic financial statements.

296. General purpose financial statements are financial statements prepared in accordance with a
financial reporting framework that is designed to
b. Meet the particular information needs of a wide range of users.
c. Meet the particular information needs of a group of users.
d. Meet the common information needs of a wide range of users.
e. Meet the common information needs of a group of users.

297. In which of the following situations would the auditor appropriately issue a standard unqualified
report with no emphasis of a matter paragraph concerning consistency?
a. A change in the method of accounting for specific subsidiaries that comprise the group of
companies for which consolidated statements are presented.
b. A change from an accounting principle that is not generally accepted to one that is generally
accepted.
c. A change in the percentage used to calculate the provision for warranty expense.
d. Correction of a mistake in the application of a generally accepted accounting principle.

298. An auditor's opinion reads as follows: "In our opinion, except for the above-mentioned limitation on
the scope of our audit..." This is an example of a(n)
a. Review opinion. c. Qualified opinion
b. Emphasis on a matter. d. Unacceptable reporting practice.

299. An auditor's report includes a statement that "the financial statements do not present fairly the
financial position in conformity with generally accepted accounting principles." This auditor's report
was probably issued in connection with financial statements that were
a. Prepared on a comprehensive basis for accounting other than GAAP.
b. Restricted for use by management.
c. Misleading.
d. Condensed.

300. If the auditor believes there is minimal likelihood that resolution of an uncertainty will have a material
effect on the financial statements, the auditor would issue a(n)
a. Qualified opinion. c. Unqualified opinion.
b. Adverse opinion. d. Disclaimer of opinion.

301. If an accounting change has no material effect on the financial statements in the current year but the
change is reasonably certain to have a material effect in later years, the change should be
a. Treated as a consistency modification in the auditor's report for the current year.
b. Disclosed in the notes to the financial statements of the current year.
c. Disclosed in the notes to the financial statements and referred to in the auditor's report for the
current year.
d. Explained in the auditors report for the current year and disclosed in the notes of the later years.

302. When comparative financial statements are presented, the fourth reporting standard, which refers to
financial statements "taken as a whole," should be considered to apply to the financial statement of
the
AT Reviewer Part III Page 38

a. Periods presented plus one preceding period.


b. Current period only.
c. Current period and those of the other periods presented.
d. Current and immediately preceding period only.

303. The client uses the last-in, first-out method to value half of its inventory and the first-in, first-out
method to value the other half. Assuming the auditor is satisfied in all other respects, under these
circumstances the auditor will issue a(n)
a. Opinion modified due to inconsistency.
b. Unqualified opinion with an emphasis of a matter middle paragraph.
c. Qualified or adverse opinion, depending on materiality.
d. Unqualified opinion.

304. Under which of the following sets of circumstances might an auditor disclaim an opinion?
a. The financial statements contain a departure from GAAP, which is material.
b. The principal auditor decides to make reference to the report of another auditor who audited a
subsidiary.
c. There has been a material change between periods in the method of the application of
accounting principles.
d. There were significant limitations on the scope of the audit.

305. An auditor includes an emphasis of a matter paragraph in an unqualified report to emphasize that the
entity being reported on is a subsidiary of another business enterprise. The inclusion of this
paragraph
a. Is appropriate and does not negate the unqualified opinion.
b. Results in the unqualified report becoming a qualified report.
c. Is a violation of generally accepted reporting standards if this information is disclosed in
footnotes to the financial statements?
d. Necessitates a revision of the opinion paragraph to include the phrase "with the foregoing
explanation."

306. An audit report should be dated as of


a. The date the report is delivered to the client.
b. The last day of fieldwork.
c. The balance sheet date of the latest period reported on.
d. The date a letter of audit inquiry is received from the entity's lawyer of record.

307. An auditor completed fieldwork on February 10, Year 2 for a December 31, Year 1 year-end client. A
significant subsequent event occurred on February 22, Year 2, which the auditor examined. Which
of the following report dates is most appropriate?
a. February 10, Year 2.
b. February 10, except for Note 1, February 22, Year 2.
c. December 31, Year 1.
d. December 31, Year 2.

308. Which of the following statements indicates a qualified opinion?


a. The financial statements do not present fairly in all material respects the financial position,
results of operations, and cash flows in conformity with GAAP.
b. The auditor does not express an opinion on the financial statements.
c. The financial statements present fairly in all material respects the financial position, results of
operations, and cash flows in conformity with GAAP.
d. Except for the effects of a matter, the financial statements present fairly in all material respects
the financial position, results of operations, and cash flows in conformity with GAAP.

309. Does an auditor make the following representations explicitly or implicitly in a standard audit report
on comparative financial statements?
Consistent application Examination of
of accounting principles evidence on a test basis
a. Explicitly Explicitly
b. Implicitly Implicitly
c. Implicitly Explicitly
d. Explicitly Implicitly

310. When management prepares financial statements on the basis of a going concern and the auditor
believes the company may not continue as a going concern, the auditor should issue
a. A qualified opinion.
b. An unqualified opinion with an emphasis of a matter paragraph.
c. A disclaimer of opinion.
d. An adverse opinion.

311. An auditor concludes that there is substantial doubt about an entity's ability to continue as a going
concern. If the entity's disclosures about continued existence are adequate, the audit report may
include
AT Reviewer Part III Page 39

A disclaimer of opinion A qualified opinion


a. Yes Yes
b. No No
c. No Yes
d. Yes No
312. The CPA receives a letter from the client's independent counsel stating that the Federal
Communications Commission is not renewing the clients broadcasting license because of alleged
irregularities. The client and its independent counsel plan to take all necessary legal action to retain
the license. However, a favorable outcome of any legal action is highly uncertain. What action
should the auditor take?
a. Issue an unqualified opinion, with an emphasis of a matter paragraph that describes the matter
which gives rise to the uncertainty.
b. Issue an unqualified opinion if full disclosure is made of the matter in a note to the financial
statements.
c. Issue an adverse opinion and disclose all reasons why.
d. Issue a piecemeal opinion with full disclosure made of the license dispute in a note to the
financial statements.

313. If the auditor believes that required disclosures are omitted from the financial statements, the auditor
should decide between issuing
a. A qualified opinion or an adverse opinion.
b. A disclaimer of opinion or a qualified opinion.
c. An adverse opinion or a disclaimer of opinion.
d. An unqualified opinion or a qualified opinion.

314. An auditor is confronted with an exception sufficiently material to warrant departing from the standard
wording of an unqualified report. If the exception relates to a departure from generally accepted
accounting principles, the auditor must decide between
a. An adverse opinion and an unqualified opinion.
b. An adverse opinion and a qualified opinion.
c. An adverse opinion and a disclaimer of opinion.
d. A disclaimer of opinion and a qualified opinion.

315. An auditor had expressed a qualified opinion on the financial statements of a prior period because
the client's financial statements departed from generally accepted accounting principles. The prior
period statements are restated in the current period to conform to generally accepted accounting
principles. The auditor's updated report on the prior period statements should
a. Express an unqualified opinion about the restated financial statements and disclose the date of
the prior report, the type of opinion expressed, the circumstances that caused a different opinion
in the prior year, and the fact that the auditors updated opinion for the prior year is now different.
b. Express an unqualified opinion about the restated financial statements and be accompanied by
the auditor's original report on the prior period.
c. Be accompanied by the auditor's original report on the prior period and bear the same date as
the auditor's original report on the prior period.
d. Qualify the opinion concerning the restated financial statements because of a change in
accounting principles.

316.Which one of the following is an example of the contents of an opinion paragraph found in an audit
report?
a. "We have audited...."
b. "Nothing came to our attention..."
c. "The financial statements do not present fairly..."
d. "An audit includes examining, on a test basis..."

317. When the financial statements contain a material departure from GAAP that the auditor believes is
justified, justification
a. Should appear in a footnote.
b. Should appear in a paragraph added before the scope paragraph.
c. Should appear in the opening paragraph.
d. Should appear in a paragraph added before the opinion paragraph.

318. Which one of the following is an instance in which the auditor would add a paragraph after the
opinion paragraph?
a. There is serious doubt that the client can continue as a going concern.
b. Management's disclosures are not adequate.
c. There are significant uncertainties that are not properly disclosed in the footnotes.
d. There is a material peso misstatement on the financial statements.

319. The opinion paragraph of the audit report for Pedring Co. states that the financial statements "do not
present fairly". Which type of audit report is this?
a. Improper. c. Disclaimer.
AT Reviewer Part III Page 40

b. Adverse. d. Qualified.

320. In which one of the following cases would an auditor issue a qualified opinion?
a. There is a highly material, and very pervasive departure from PAS 8 and PAS 1.
b. There is a change in accounting principles promulgated by the FRSC.
c. There is an immaterial peso misstatement on the financial statements.
d. There is a material departure from GAAP that is centered around two accounts.

321. A client company has a history of negative cash flow trends and continuing losses. Which type of
opinion will the auditor most likely give?
a. Adverse. c. Qualified.
b. Unqualified with emphasis of a matter language. d. Disclaimer of opinion.

322. In which one of the following instances would an auditor issue an adverse opinion?
a. Management declines to present earnings per share in the income statement.
b. There is substantial doubt about the entity's ability to continue as a going concern.
c. There is a material peso misstatement that overshadows the entire financial statement.
d. The client does not allow the auditor to send confirmations to its three largest clients.

323. In which one of the following instances would an auditor most likely issue a disclaimer of opinion?
a. Management will not sign a management representation letter.
b. Management declines to provide a statement of cash flow.
c. The auditor is independent of the client.
d. The auditor is unable to confirm receivables but performs alternative procedures.

324. In which one of the following instances would an auditor most likely issue a standard unqualified
opinion?
a. Management's disclosures are missing or inadequate.
b. There is substantial doubt about the entity's ability to continue as a going concern.
c. There is a significant limitation on the scope of the engagement.
d. There is an immaterial deviation from GAAP related to capitalizing repairs.

325. In which one of the following instances would an auditor not issue a disclaimer of opinion?
a. The auditors are not invited to the periodic inventory at year end.
b. There are significant misstatements in the financial statements.
c. There is a significant limitation on the scope of the engagement.
d. There is insufficient evidence for the auditor to form an opinion on the fairness of the financial
statements.

326. The division of responsibility between the reporting company's management and the auditor firm is
set forth in the
a. Auditors responsibility paragraph. c. Notes to the financial statements.
b. Introductory paragraph d. Opinion paragraph.

327 When financial statements contain a material, unjustified departure from GAAP, the audit report
should contain An emphasis of a matter paragraph
A qualification after the opinion paragraph
a. Yes No
b. Yes Yes
c. No Yes
d. No No

328. When the auditor wishes to emphasize a matter in the financial statements, the audit report should
contain
An emphasis of a matter paragraph
A qualification after the opinion paragraph
a. Yes No
b. Yes Yes
c. No Yes
d. No No

329. When the auditor is unable to obtain sufficient competent evidence because the client did not allow a
procedure to be completed, the report will most likely contain
An emphasis of a matter paragraph
A qualification after the opinion paragraph
a. Yes No
b. Yes Yes
c. No Yes
d. No No

330. When the financial statements do not contain a footnote the auditor believes is necessary for fair
presentation, the failure to include the footnote is considered to be
a. A scope limitation.
AT Reviewer Part III Page 41

b. An uncertainty.
c. A departure from GAAP.
d. An act discreditable.

331. When an auditor is faced with a material departure from GAAP that is pervasive, the audit report
should contain
a. An unqualified opinion.
b. A qualified opinion with an emphasis of a matter paragraph.
c. An adverse opinion.
d. A disclaimer of opinion.

332. If the auditor believes that there is a remote probability that resolution of an uncertainty will have a
material effect on the financial statements, the auditor would issue
a A disclaimer of opinion.
b. A standard unqualified opinion.
c. An adverse opinion.
d. An unqualified opinion with emphasis of a matter paragraph.

333. When an auditor lacks independence with respect to a client, the auditor should issue
a. A disclaimer of opinion.
b. An adverse opinion.
c. A qualified opinion with emphasis of a matter paragraph.
d. An unqualified opinion.

334 . Morgan, CPA, suspects that a pervasive scheme of illegal bribes exists throughout the operations of
Atlas Import-Export, Inc., a new audit client. Morgan notified the audit committee and Atlas legal
counsel, but neither could assist Morgan in determining whether the amounts involved were material
to the financial statements or whether senior management was involved in the scheme. Under these
circumstances, Morgan should
a. Express an unqualified opinion with a separate emphasis of a matter paragraph.
b. Disclaim an opinion on the financial statements.
c. Express an adverse opinion on the financial statements.
d. Issue a special report regarding the illegal bribes.

335. After issuing a report, an auditor concludes that an auditing procedure considered necessary at the
time of the audit was omitted from the audit. The auditor should first
a. Undertake to apply the omitted procedure or alternative procedures that would provide a
satisfactory basis for the auditors opinion
b. Assess the importance of the omitted procedure to the auditors ability to support the opinion
expressed on the financial statements taken as a whole
c. Notify the audit committee or the board of directors that the audit opinion can no longer be relied
upon
d. Review the results of other procedures that were applied to compensate for the one omitted or to
make its omission less important

336. The auditor concludes that there is a material inconsistency in the other information in an annual
report to shareholders containing audited financial statements. If the client refuses to revise or
eliminate the material inconsistency, the auditor should
a. Revise the auditors report to include a separate explanatory paragraph describing the material
inconsistency
b. Consult with a party whose advice might influence the client, such as the clients legal counsel
c. Issue a qualified opinion after discussing the matter with the clients board of directors
d. Consider the matter closed since the other information is not in the audited financial statements

337. An auditor would be most likely to consider expressing a qualified opinion if the clients financial
statements include a footnote on related party transactions that
a. Lists the amounts due from related parties including the terms and manner of settlement
b. Discloses compensating balance arrangements maintained for the benefit of related parties
c. Represents that certain transactions with related parties were consummated on terms equally as
favorable as would have been obtained in transactions with unrelated parties
d. Presents the dollar volume of related party transactions and the effects of any change in the
method of establishing terms from that of the prior period

338. If the auditor decides to draw attention to large related party transactions occurring in the financial
statements of the client, which report will most likely be issued?
a. Qualified.
b. Unqualified with an emphasis of a matter paragraph
c. Adverse.
d. Consolidation.

Related Reporting Issues


AT Reviewer Part III Page 42

339. Comfort letters ordinarily are:


Addressed to the client's Signed by the client's
a. Audit committee Independent auditor
b. Underwriter of securities Senior management
c. Audit committee Senior management
d. Underwriter of securities Independent auditor

340. Which of the following matters is covered in a typical comfort letter?


a. Negative assurance concerning whether the entitys internal control activities operated as
designed during the period being audited.
b. An opinion regarding whether the entity complied with laws and regulations under Government
Auditing Standards.
c. Positive assurance concerning whether unaudited condensed financial information complied with
generally accepted accounting principles.
d. An opinion as to whether the audited financial statements comply in form with the accounting
requirements of the SEC.

341. When an accountant issues to an underwriter a comfort letter containing comments on data that have
not been audited, the underwriter most likely will receive
a. Positive assurance on supplementary disclosures
b. Negative assurance on capsule information
c. A disclaimer on prospective financial statements
d. A limited opinion of pro forma financial statements

342. When audited financial statements are presented in a clients document containing other information, the
auditor must
a. Perform inquiry and analytical procedures to ascertain whether the other information is reasonable
b. Add an explanatory paragraph to the auditors report without changing the opinion on the financial
statements
c. Perform the appropriate substantive auditing procedure to corroborate the other information
d. Read the other information to determine that it is consistent with the audited financial statements

343. An auditor concludes that there is a material inconsistency in the other information in an annual report to
shareholders containing audited financial statements. However, the auditor has not performed
procedures to corroborate the other information. If the auditor concludes that the financial statements do
not require revision, but the client refuses to revise or eliminate the material inconsistency, the auditor
may
a. Revise the auditors report to include a separate explanatory paragraph describing the material
inconsistency
b. Express a qualified opinion after discussing the matter with the clients directors
c. Consider the matter closed because the other information is not in the audited statements
d. Disclaim an opinion on the financial statements after explaining the material inconsistency in a
separate paragraph

PAPS 1000 Inter-Bank Confirmation Procedures

344. Financial and business relationships which require bank confirmation include the following:

Relationships between: a b c d
The bank and other banks within the same country Yes Yes Yes Yes
The bank and other banks in different countries Yes No Yes No
The bank and its non-bank customers No Yes Yes No

PAPS 1004 The Relationship Between (Banking Supervisors) BSP and Banks External Auditors

345 PAPS 1004, on which PAPS 1004 is based, was prepared in association with which of the following?
a. International Accounting Standards Board
b. Basel Committee on Banking Supervision (the Basel Committee)
c. Professional Regulatory Board of Accountancy
d. Professional Regulation Commission

346. Which of the following is an element of prudential supervision?


a. Minimum capital requirements
b. Supervisory review process
c. Market discipline
d. All of these are elements or pillars of prudential supervision

347. This is the most significant of banking risks, in terms of historical loss experience
a. Credit risk c. Operational risk
AT Reviewer Part III Page 43

b. Market risk d. Information risk

PAPS 1005 The Special Considerations in the Audit of Small Business (Small Entity)

348. For purposes of PAPS, a small entity is an entity where there is a concentration of ownership and
management in a small number of individuals (often a single individual) and one or more of the
following are found:
a. Few sources of income
b. Unsophisticated record-keeping
c. Limited internal controls together with the potential for management override of controls.
d. All of these.

PAPS 1006 Auditing Financial Statements of Banks

349. For purposes of PAPS 1006, a bank is


a. A type of financial institution whose principal activity is the taking of deposits and borrowing for
the purpose of lending and investing and that is recognized as a bank by the regulatory
authorities in any country in which it operates.
b. A type of institution whose peripheral activity is the taking of deposits and borrowing for the
purpose of lending and investing and that is recognized as a bank by the regulatory authorities in
any country in which it operates.
c. A type of financial institution whose principal activity is the taking of deposits and borrowing for
the purpose of lending and investing and that is recognized as a bank by the regulatory
authorities in a limited number of areas which it operates.
d. A type of financial institution whose principal activity is lending and investing and that is
recognized as a bank by the regulatory authorities in any country in which it operates.
PAPS 1013 Electronic Commerce (Effect on the Audit of Financial Statements)

350. Which is considered an operating system?


a. A program that generates text in a presentable form such as a letter or memo.
b. A program that commands and controls other programs and the hardware.
c. A program that performs mathematical calculations in cells and produces charts.
d. A system of transmitters and silicon wired together sufficient to carry out commands and
mathematic functions.

351. Which of the following is the primary reason that many auditors hesitate to use embedded audit
modules?
a. Embedded audit modules cannot be protected from computer viruses.
b. Auditors are required to monitor embedded audit modules continuously to obtain valid results.
c. Embedded audit modules can easily be modified through management tampering.
d. Auditors are required to be involved in the system design of the application to be monitored.

352. The auditor is required by the standard to consider CIS environment in designing audit procedures to
reduce risk to an acceptably low level. Which of the following statements is incorrect?
a. The auditors specific audit objectives do not change whether financial information is processed
manually or by the computer.
b. The methods of applying audit procedures to gather audit evidence are not influenced by
methods of computer processing.
c. The auditor may use either manual audit procedures, computer-assisted audit techniques
(CAATs), or a combination of both to obtain sufficient appropriate audit evidence.
d. In some CIS environments, it may be difficult or impossible for the auditor to obtain certain data
for inspection, inquiry, or confirmation without the aid of a computer.

352. An auditor who wishes to capture an entity's data as transactions are processed and continuously
test the entity's computerized information system most likely would use which of the following
techniques?
a. Snapshot application.
b. Embedded audit module.
c. Integrated data check.
d. Test data generator.

353. Which of the following is an engagement attribute for an audit of an entity that processes most of its
financial data in electronic form without any paper documentation?
a. Discrete phases of planning, interim, and year-end fieldwork.
b. Increased effort to search for evidence of management fraud.
c. Performance of audit tests on a continuous basis.
d. Increased emphasis on the completeness assertion.

354. Which of the following strategies would a CPA most likely consider in auditing an entity that
processes most of its financial data only in electronic form, such as a paperless system?
a. Continuous monitoring and analysis of transaction processing with an embedded audit module.
b. Increased reliance on internal control activities that emphasize the segregation of duties.
AT Reviewer Part III Page 44

c. Verification of encrypted digital certificates used to monitor the authorization of transactions.


d. Extensive testing of firewall boundaries that restrict the recording of outside network traffic.

355. When an auditor tests a computerized accounting system, which of the following is true of the test
data approach?
a. Several transactions of each type must be tested.
b. Test data are processed by the client's computer programs under the auditor's control.
c. Test data must consist of all possible valid and invalid conditions.
d. The program tested is different from the program used throughout the year by the client.

356. Which of the following statements most likely represents a disadvantage for an entity that keeps
microcomputer-prepared data files rather than manually prepared files?
a. Attention is focused on the accuracy of the programming process rather than errors in individual
transactions.
b. It is usually easier for unauthorized persons to access and alter the files.
c. Random error associated with processing similar transactions in different ways is usually
greater.
d. It is usually more difficult to compare recorded accountability with physical count of assets.

357 In the preliminary survey the auditor learns that a department has several microcomputers. Which of
the following is usually true and should be considered in planning the audit?
a. Microcomputers, though small, are capable of processing financial information, and physical
security is a control concern.
b. Microcomputers are limited to applications such as worksheet generation and do not present a
significant audit risk.
c. Microcomputers are generally under the control of the data processing department and use the
same control features.
d. Microcomputers are too small to contain any built-in control features. Therefore, other controls
must be relied upon.

358. When an on-line, real-time (OLRT) computer-based processing system is in use, internal control
can be strengthened by
a. Providing for the separation of duties between keypunching and error listing operations.
b. Attaching plastic file protection rings to reels of magnetic tape before new data can be
entered on the file.
c. Making a validity check of an identification number before a user can obtain access to the
computer files.
d. Preparing batch totals to provide assurance that file updates are made for the entire input.

359. Given the increasing use of microcomputers as a means for accessing data bases, along with on-
line real-time processing, companies face a serious challenge relating to data security. Which of the
following is not an appropriate means for meeting this challenge?
a. Institute a policy of strict identification and password controls housed in the computer software
that permit only specified individuals to access the computer files and perform a given function.
b. Limit terminals to perform only certain transactions.
c. Program software to produce a log of transactions showing date, time, type of transaction, and
operator.
d. Prohibit the networking of microcomputers and do not permit users to access centralized data
bases.
360. In parallel simulation, actual client data are reprocessed using an auditor software program.
An advantage of using parallel simulation, instead of performing tests of controls without a computer,
is that:
a. The test includes all types of transaction errors and exceptions that may be encountered.
b. The client's computer personnel do not know when the data are being tested.
c. There is no risk of creating potentially material errors in the client's data.
d. The size of the sample can be greatly expanded at relatively little additional cost.

361. When an auditor tests the internal controls of a computerized accounting system, which of the
following is true of the test data approach?
a. Test data are coded to a dummy subsidiary so they can be extracted from the system under
actual operating conditions.
b. Test data programs need not be tailor-made by the auditor for each client's computer
applications.
c. Test data programs usually consist of all possible valid and invalid conditions regarding
compliance with internal controls.
d. Test data are processed with the client's computer and the results are compared with the
auditor's predetermined results.

362. When companies use information technology (IT) extensively, evidence may be available only in
electronic form. What is an auditor's best course of action in such situations?
a. Assess the control risk as high.
b. Use audit software to perform analytical procedures.
c. Use generalized audit software to extract evidence from client databases.
AT Reviewer Part III Page 45

d. Perform limited tests of controls over electronic data.

363. Company A has recently converted its manual payroll to a computer-based system. Under the old
system, employees who had resigned or been terminated were occasionally kept on the payroll and
their checks were claimed and cashed by other employees, in collusion with shop foremen. The
controller is concerned that this practice not be allowed to continue under the new system. The best
control for preventing this form of "payroll padding" would be to
a. Conduct exit interviews with all employees leaving the company, regardless of reason.
b. Require foremen to obtain a signed receipt from each employee claiming a payroll check.
c. Require the human resources department to authorize all hires and terminations, and to forward
a current computerized list of active employee numbers to payroll prior to processing. Program
the computer to reject inactive employee numbers.
d. Install time clocks for use by all hourly employees.

364. Which one of the following is not a relevant factor in assessing the control risk of a computerized
environment?
a. Rapid change in computerized environments requires management to institute effective methods
to control those systems.
b. Effective controls over computer systems are necessary to compete effectively in a global
marketplace.
c. An auditor's objective for assessing control risk is the same in a computerized system as it is in a
manual system.
d. An auditor's method of testing the effectiveness of the system controls is the same in a
computerized system as in a manual system.

365. Processing data through the use of simulated files provides an auditor with information about the
operating effectiveness of control policies and procedures. One of the techniques involved in this
approach makes use of
a. Controlled reprocessing
b. An integrated test facility
c. Input validation
d. Program code checking

366.Totals of amounts in computer-record data fields which are not usually added for other purposes but
are used only for data processing control purposes are called
a. Record totals.
b. Hash totals.
c. Processing data totals.
d. Field totals.

367. A hash total of employee numbers is part of the input to a payroll master file update program. The
program compares the hash total to the total computed for transactions applied to the master file.
The purpose of this procedure is to:
a. Verify that employee numbers are valid.
b. Verify that only authorized employees are paid.
c. Detect errors in payroll calculations.
d. Detect the omission of transaction processing.

368. Which of the following strategies would a CPA most likely consider in auditing an entity that
processes most of its financial data only in electronic form, such as a paperless system?
a. Continuous monitoring and analysis of transactions processing with an embedded audit module
b. Increased reliance on internal control activities that emphasize the segregation of duties
c. Verification of encrypted digital certificates used to monitor the authorization of transactions
d. Extensive testing of firewall boundaries that restrict the recording of outside network traffic

369. One common type of CAAT is the use of audit software to process data of audit significance from the
entitys information system. An audit software that has widespread popularity because it is easy to
use and requires little computer background on the part of the auditor; it can be used both
mainframe and PC systems; it allows auditor to perform his/her tests independent of the entitys
computer processing personnel; and it can be used to audit data in most file formats and structures
is called a
a. Customized program.
b. Purpose-written program.
c. Utility program.
d. Package or generalized audit software (GAS).

369. Which of the following procedures would the principal auditor most likely perform after deciding to
make reference to another CPA who audited a subsidiary of the entity?
a. Review the working papers and the audit programs of the other CPA
b. Visit the other CPA and discuss the results of the other CPAs audit procedures
c. Make inquiries about the professional reputation and independence of the other CPA
AT Reviewer Part III Page 46

d. Determine that the other CPA has a sufficient understanding of the subsidiarys internal control

370. Software that organizes a collection of data into logically similar records and files is most commonly
referred to as
a. Access control software. c. Database management system software.
b. Operating system software. d. Application software.

371. Which of the following strengthens segregation of duties in the electronic environment.
a. The computer operator should not have access to programs or data except for authorized
purposes.
b. Programmers test new developments on live data.
c. Database administrators are allowed to maintain the data.
d. Internal auditors are not allowed to test changes to software.

372. The combination of which functions in an information technology environment are not necessarily a
control weakness?
a. User and programming functions
b. Database administrator and data input functions
c. Programmers and authorization of changes
d. Users and certain data access

373. General controls address all of the following except


a. Planning and controlling the information technology department.
b. Input of data into the payroll application.
c. Controlling access to accounting programs.
d. Maintaining application servers.

374. The control structure over program changes should include control procedures that are sufficient to
ensure that:
a. Changes to applications are not approved in advance.
b. Changes are made by the users to computer applications.
c. The latest version of the application is sent to production.
d. Changes are tested and documented after implementation.

375. Which one of the following is not a fundamental control concept in evaluating the organization of
data processing?
a. The authorization for all transactions should originate outside the information technology
department.
b. There should be a fundamental segregation of duties between users and information technology
departments.
c. The programming department should not have access the code under development.
d. Access to data should be provided only to users authorized by the data's owner.

376. Which one of the following is not a fundamental control concept in evaluating the organization of
data processing?
a. The authorization for all transactions should originate outside the information technology
department.
b. There should be a fundamental segregation of duties between users and the information
technology departments.
c. Management should periodically evaluate the information system function for operational
efficiency, integrity, security, and consistency with organizational objectives.
d. The external auditors should periodically audit applications and operations.

377. Which one of the following is the least likely to be used today to authenticate the user of a computer
system?
a. Username and password c. Biometrics
b. Token card d. Both B and C.

378. Auditors review the adequacy of client's documentation of accounting information systems to do all of
the following except
a. Determine computer assisted audit techniques that may be utilized.
b. Assess control risk associated with the information technology environment.
c. Rewrite the coding used to generate programs to satisfy audit assertions.
d. Understand the use of technology and systems to determine the audit procedures to be
performed.

379. Which of the following is correct regarding data access control?


a. Access to data should be shared company-wide in an effort to integrate information.
b. All users should be allowed to change data with appropriate audit logs.
c. A monitoring system should be instituted to monitor external hackers.
AT Reviewer Part III Page 47

d. Only individuals having a justified reason to view data should be allowed access.

380. What is the primary purpose of hash totals?


a. Batch integrity c. Data validation.
b. Transmission confidentiality. d. Communication repudiation.

381. Backup controls, used to prevent data and software loss would include procedures to:
a. Load balance in the event of heavy traffic across the network.
b. Constantly monitor the network for intrusion and attack.
c. Fail-over to a mirrored storage device in the event of interruption or loss.
d. Both B and C.

382. A firewall is best known as a(n):


a. Single source of Internet security. c. Intrusion detection system.
b. Layer of a multi-layered security plan. d. Demilitarized zone.

383. Which of the following is considered a batch control?


a. Counting the total number of records input and comparing them to the system.
b. An identifier added to the field of the record that is unique to the transaction.
c. Audit logging capabilities to determine which accounting personnel performed a transaction.
d. A mathematical algorithm used to verify the validity of data.

384. Which one of the following is an example of an edit test for limits?
a. Only numbers are allowed to be input into the social security number field and dashes are pre-
formatted.
b. Each sales order is given an identifier that is also tagged to the receipt of goods record, the
customer invoice record and the cash receipts record.
c. The purchasing department may order materials up to P5,000 in total before a second electronic
authorization is required.
d. Supplies may not be purchased from any vendor not authorized and existing in the vendor table.

385. The control totals used to ensure the completeness of processing in a batch processing environment
include
a. Record count. c. Hash totals.
b. Control totals. d. All of the following

386. A database is relational and is programmed to call up the customer shipping address into the order
entry interface when a repeat customer calls for a new order. This allows the order entry personnel
to verify that the shipping address remains valid and is an example of a(n):
a. Batch control. c. Reasonable range control.
b. On-line processing control. d. Self-checking control.

387. The documents and records that allow a user or auditor to trace a transaction from its origination
through its final disposition are referred to as
a. Authentication. c. An audit trail.
b. Callback. d. Echo check.

388. Which of the following is considered a monitoring control in the electronic environment?
a. Reconciliation of control totals.
b. Test data to ensure calculations are performed as expected.
c. Computer logs of attempted penetration.
d. Logical screen layout.

389 An auditor sends fictitious information through the system for processing. This is an example of
a. The test data approach. c. Generalized audit software.
b. A tagging and tracing approach. d. Audit procedures using manual efforts.

390 An internal auditor creates a dummy division of the organization and sends test transactions through
the system along with valid transactions. This is an example of
a. An integrated test facility. c. Generalized audit software
b. A tagging and tracing approach. d. Audit procedures using manual efforts.

391. An internal auditor selects a random transaction and examines it through the processing routine.
This activity is an example of:
a. An integrated test facility. c. Generalized audit software
b. A tagging and tracing approach. d. Substantive audit procedures using electronic
means

392.An auditors use of electronic means to foot a large subsidiary ledger or to select a sample is an
example of the utilization of
a. An integrated test facility. c. Generalized audit software
b. A tagging and tracing approach. d. Risk assessment using electronic means.
AT Reviewer Part III Page 48

393. An auditor desires to test all write-offs of accounts receivable during the period exceeding P10,000
for proper authorization. The population of write-offs is found in a table of transactions in the
database and is voluminous. To be efficient, the auditor will most likely use what approach to
selecting the sample as described?
a. Examination of exception reports.
b. Examination of a report of all write-offs in chronological order.
c. Use of limit tests to perform audit testing.
d. Use of generalized audit software to list all items fitting the criteria.

394. An example of the use of generalized audit software for the testing of balances is:
a. Tracing the cash on the bank reconciliation to the bank statement balance.
b. Selecting a random sample of customer invoices to confirm.
c. Preparation of management representation letters.
d. Creation of dummy companies for appropriate evaluation of posting.

395. A tool that may be used to analyze patterns such as the assignment of invoice numbers or amounts
to fictitious invoices is:
a. Generalized audit software c. Lead generation software.
b. Paperless audit software. d. Integrated test facility software

396 Tagging and tracing might best be used to test which of the following?
a. The number of confirmations returned compared to the number of confirmations sent out.
b. The corporate credit card is only being used for business transactions.
c. All customer orders shipped were also invoiced and that cash was received.
d. Quantities of product F411029 have been accurately counted in the warehouse.

397. Which of the following is not a component of an electronic audit trail?


a. Validity testing of transaction.
b. Location from which the transaction originated.
c. Individual responsible for the transaction.
d. Date and time of transaction.

398. Which of the following is not an example of input validation (edit) tests?
a. Alphanumeric. c. Limit
b. Continuity. d. Validity

399. Which of the following is not one of the seven fundamental control concepts in evaluating the
organization of the data processing department?
a. Access to data is provided only to authorized users.
b. Data processing is responsible for all custodial functions associated with data, data files,
software and related documentation.
c. Data processing is responsible for authorizing, reviewing and testing all applications
developments and changes in programs.
d. Management should periodically evaluate the information system function for operational
efficiency, integrity, and security.

400. Which of the following is not a required segregation of duties within a data processing function?
a. Data processing personnel should not have access to programs or data except when authorized.
b. Users should review and test all significant computer program changes.
c. The computer program librarian should have limited access to the computer.
d. All are required of data processing function.

*********************************

You might also like